Math Sec - Diploma - Module.. - Hamachila PDF

You might also like

Download as pdf or txt
Download as pdf or txt
You are on page 1of 148

COLLEGE MATHEMATICS MADE SIMPLE

BY

MR. HAMACHILA. I
UNIVERSITY OF ZAMABIA
IN ASSOCIATION WITH

MONZE COLLEGE OF EDUCATION

Mathematics may not teach us how to Add happiness or how to SUBTRACT


sadness. But, it does teach one important thing. Every PROBLEM has a solution.

Page 1 | H A M A C H I L A . I
SETS
Definition: A set is a collection of distinct and well defined objects.

PRESENTATION OF A SET
There are basically four ways of presenting a set:
 Listing
A set of the first six whole numbers can be listed as follows: A= {0, 1, 2, 3, 4, 5}
 Describing
A set of the first six whole numbers, A = {first six whole numbers}
 Set builder notation

A = {x: 0 ≤ x ≤ 5, x ∈ W}
 Diagrammatically
In a Venn diagram, set A, the first seven whole numbers can be presented in the following way:

𝐴
𝐸
.5 .4
.2 .3 .1.0
.0 .6
.3 .1

FINITE AND INFINITE SETS


A set is finite if its elements can be listed. For example, a set of vowels in English alphabet can be listed as
A = {a, e, i, o, u}.
A set is infinite if it is impossible to list all the members. In a set of Natural numbers, it is impossible to list

all the members of this set. We usually list such a set as ℕ = {1, 2, 3 …} where the three dots (…) mean
“and so on”.

EQUAL AND EQUIVALENT SETS


Equal Sets: Two sets A and B are said to be equal if and only if they contain exactly the same elements.
This implies that every member of A is a member of B and every member of B is a member of A. The
members are in one-to-one correspondence. For example if P = {r, s, t, u, v, w} and Q = {u, s, w, t, v, r},
then set P = set Q because all the elements of set P are also found in set Q, without any shortfalls or extra
elements in any set. Therefore, P = Q.

Page 2 | H A M A C H I L A . I
Equivalent Sets: Two or more sets are said to be equivalent if they contain the same number of elements
which could not necessarily be identical. For instance, if X = {1, 2, 3, 4, 5} and Y= {v, w, x, y, z} then set

X and set Y are equivalent sets, i.e. X⇔Y. Both sets have five elements each. Note that all equal sets are
equivalent sets, but not all equivalent sets are necessarily equal sets.

DISJOINT SETS
Two sets A and B are said to be disjoint sets if and only if their intersection is empty. That is, A  B  
Example: Let U = the set of positive counting numbers, be the Universal set, and A = the set of even
numbers and B = be the set of odd numbers. Find A  B ?
Solution We know that there is no number which is both even and odd, so A  B  

SUBSETS
Definition: A set A is a subset of a set B, written, A  B , if every element of A is also an element of B.
Stop and think: are all empty sets equal? If you are confused, get back to the definition and try to read it
again.
Proper and improper subsets
The list of subsets will always include the empty set and the set itself. These two sets are called improper
subsets while the remaining subsets are proper subsets. A proper subset is a set which contains at least one
element but not all the elements of the set.
Note that an empty set has only one subset and that is the empty set itself.
Example:
Given that set X = {a, b, c} list all the subsets of X and the improper subsets. The subsets are { }, {a},
{b}, {c}, {a, b}, {a, c}, {b, c}, and {a, b, c}. The first, { } and last {a, b, c} subsets listed here are improper
subsets while the rest are proper subsets.

Number of subsets = 2ⁿ, where n is the number of elements in a given set. For example; find the number of
subsets in P = {a, b, c, d}.
Number of subsets = 2ⁿ
= 2⁴ (n = 4 elements)
=2x2x2x2
= 16
ACTIVITY
Now try the next two questions. Read and understand the questions before you attempt to answer them.

Page 3 | H A M A C H I L A . I
1. Find the number of subsets for a set which has six elements. Try to think
through the steps as you calculate, after you have got your answer, now try the
next question.

2. If a set has 128 subsets, how many elements are in that set?

Check your solutions

The first one is given by No. of subsets = 2ⁿ


= 2n (n = 6 elements)
= 26
=2x2x2x2x2x2
= 64

The second one allows you to move from the answer to the number of elements.
128 = 2ⁿ
2ⁿ = 27

∴ n=7
This means that the set has 7 elements in it. You can also find the answer by continuously dividing 128
by 2 until you get 1 just like we did in number bases.

NUMBER OF ELEMENTS IN A SET


The number of distinct elements in 𝐵 is written as 𝑛(𝐵). This is sometimes known as the cardinal number
of the set. The symbol # is used to denote this cardinal number. If 𝐵 = {2, 4, 6, 8, 10} the number of
elements in 𝐵 is 5. This is written as 𝑛(𝐵) = 5 𝑜𝑟 #𝐵 = 5.
Activity
If 𝐾 = {𝑥: ­3 ≤ 𝑥 < 5, 𝑥 ∈ 𝑍}, find the 𝑛(𝐾)

OPERATIONS ON SETS.
Notations
 = union,  = intersection, A' = A compliment, which means all elements in the universal set except
those in A,  = empty set or   = empty set. We use one of these two symbols and not both.

Page 4 | H A M A C H I L A . I
Just as numbers are added, subtracted, sets can also be operated under union, intersection and complements
of sets between two or more sets.
Intersection of sets: Notation: A  B , reads A intersection B

Definition: Intersection: is a set of elements which are found in both set A and set B.
A  B is a set of elements found in both A and B

The intersection of sets A and B contains a particular group of elements that are found in both set A and
in set B.
Union of sets: Notation; A B read, A union B
Definition: Union is a set of elements which are found in A or B or in both A and B ) note that when
writing the elements of the union of two or more sets, each element is written once even if it appears in
both or in more sets.
A  B is a set of elements which are found in A or in B or in both A and B ( note that elements found in
both A and B are written once).

Page 5 | H A M A C H I L A . I
Complement: Notation; A ' reads, A complement

Definition: A ' is the set of elements in the universal set except elements in A.

A' (A-dash) is called the complement of A. It contains all elements which are in the universal set but not
members of A. A and 𝐴′ added together make up the Universal set. That is A  A'  U
Example 1.3.3
Let u  a, e, i, o, u and subsets A  a, e, u and B  a, i, o

Find
(i) A B (ii) A B (iii) A'B'
(iv) ( A  B)'
Solutions
(i) List the elements found in both A and B
A  B  a , note that a is the only element found in both A and B

Page 6 | H A M A C H I L A . I
(ii) Here we list all the elements found in A and those elements found in B
A  B  a, e, i, o, u , note that ‘a’ which appears in both A and B it is written once.

(iii) List the elements of A’ and B’ and write the elements found in both A’ and B’
A'  i, o and B' e, u then A' B'    means there is no element common in A’ and B’.

(iv) List the elements found in A and B then find the complement of this set.
A  a, e, u and B  a, i, othen, A  B  a, e, i, o, u , ( A  B)'  

DIFFERENCE OF SETS
The difference of sets A and B denoted by A – B is the set of all those elements of A which are not members
of B. The difference of sets A and B is also denoted by A\B (also read as A less B)

i.e. A – B = {x ∈ E: x ∈ A and x ∉ B}

= {x : x ∈ A and x ∈ B'}

= {x : x ∈ (A ∩ B')}
= A ∩ B'

Page 7 | H A M A C H I L A . I
Therefore, A – B = A ∩ B'
Also B – A = B ∩ A'
Generally, A – B ≠ B – A . Set difference is NOT commutative.
Set difference can be represented by shading in Venn diagrams as follows:

A\B B\A
Example
Let E= {a, b, c, d, e, f, g, h, i, j}, A {a, c, d, h, j} and B = {b, c, f, h, i, j}
A' = {b, e, f, g, i}
B' = {a, d, e, g}
B ∩ A'= {b, f, i}
A ∩ B' = {a, d}
A – B = {a, d}
B – A = {b, f, i}
From the above we can see that:
(i) A – B = {a, d}
A ∩ B' = {a, d}

∴ A – B = A ∩ B’

(ii) B – A = {b, f, i}
B ∩ A' = {b, f, i}

∴ B – A = B ∩ A’
LAWS OF THE ALGEBRAIC OF SETS
1. a) A  A  A b. A A  A Idempotent laws
2 a) ( A  B)  C  A  ( B  C ) b. ( A  B)  C  A  ( B  C ) Associative laws
3 a) A  B  B  A b. A B  B  A Commutative laws
4 a) A  ( B  C )  ( A  B)  ( A  C )

b) A  ( B  C )  ( A  B)  ( A  C ) Distributive laws

Page 8 | H A M A C H I L A . I
5 a) A    A b) A U  A Identity laws

6. a) A  U  U b) A   A Identity laws

7 a) A  A  U
'
b. A  A'   Compliment laws

8 a) ( A )  A
' '
b. U '   , ' U Compliment laws

a) ( A  B)  A  B ( A  B)  A '  B '
' '
9 b. De Morgan’s laws
NOTE: These laws are very important and useful, make sure you know them. Always examinable,
especially on set distributions and simplifications. Also notice how compliment behaves when it meets
other operations in brackets.

De Morgan’s law:
The complement of the union of two sets is equal to the intersection of their complements and the
complement of the intersection of two sets is equal to the union of their complements. These are called De
Morgan’s laws.
For any two finite sets A and B;
(i) (A U B)' = A' ∩ B' (which is a De Morgan's law of union).
(ii) (A ∩ B)' = A' U B' (which is a De Morgan's law of intersection).

Proof of De Morgan’s law: (A U B)' = A' ∩ B'


Let P = (A U B)' and Q = A' ∩ B'
Let x be an arbitrary element of P then x ∈ P ⇒ x ∈ (A U B)'
⇒ x ∉ (A U B)
⇒ x ∉ A and x ∉ B
⇒ x ∈ A' and x ∈ B'
⇒ x ∈ A' ∩ B'
⇒x∈Q
Therefore, P ⊂ Q …………….. (i)
Again, let y be an arbitrary element of Q then y ∈ Q ⇒ y ∈ A' ∩ B'
⇒ y ∈ A' and y ∈ B'
⇒ y ∉ A and y ∉ B
⇒ y ∉ (A U B)
⇒ y ∈ (A U B)'
⇒y∈P

Page 9 | H A M A C H I L A . I
Therefore, Q ⊂ P …………….. (ii)
Now combine (i) and (ii) we get; P = Q i.e. (A U B)' = A' ∩ B'

Proof of De Morgan’s law: (A ∩ B)' = A' U B'


Let M = (A ∩ B)' and N = A' U B'
Let x be an arbitrary element of M then x ∈ M ⇒ x ∈ (A ∩ B)'
⇒ x ∉ (A ∩ B)
⇒ x ∉ A or x ∉ B
⇒ x ∈ A' or x ∈ B'
⇒ x ∈ A' U B'
⇒x∈N
Therefore, M ⊂ N …………….. (i)
Again, let y be an arbitrary element of N then y ∈ N ⇒ y ∈ A' U B'
⇒ y ∈ A' or y ∈ B'
⇒ y ∉ A or y ∉ B
⇒ y ∉ (A ∩ B)
⇒ y ∈ (A ∩ B)'
⇒y∈M
Therefore, N ⊂ M …………….. (ii)
Now combine (i) and (ii) we get; M = N i.e. (A ∩ B)' = A' U B'

Examples on De Morgan’s law:


1. If U = {j, k, l, m, n}, X = {j, k, m} and Y = {k, m, n}.
Proof of De Morgan's law: (X ∩ Y)' = X' U Y'.
Solution:
We know, U = {j, k, l, m, n}
X = {j, k, m}
Y = {k, m, n}
(X ∩ Y) = {j, k, m} ∩ {k, m, n}
= {k, m}

Therefore, (X ∩ Y)' = {j, l, n} ……………….. (i)


Again, X = {j, k, m} so, X' = {l, n}

Page 10 | H A M A C H I L A . I
and Y = {k, m, n} so, Y' = {j, l}
X' ∪ Y' = {l, n} ∪ {j, l}

Therefore, X' ∪ Y' = {j, l, n} ……………….. (ii)


Combining (i)and (ii) we get;
(X ∩ Y)' = X' U Y'. Proved

2. Let U = {1, 2, 3, 4, 5, 6, 7, 8}, P = {4, 5, 6} and Q = {5, 6, 8}.


Show that (P ∪ Q)' = P' ∩ Q'.
Solution:
We know, U = {1, 2, 3, 4, 5, 6, 7, 8}
P = {4, 5, 6}
Q = {5, 6, 8}
P ∪ Q = {4, 5, 6} ∪ {5, 6, 8}
= {4, 5, 6, 8}
Therefore, (P ∪ Q)' = {1, 2, 3, 7} ……………….. (i)
Now P = {4, 5, 6} so, P' = {1, 2, 3, 7, 8}
and Q = {5, 6, 8} so, Q' = {1, 2, 3, 4, 7}
P' ∩ Q' = {1, 2, 3, 7, 8} ∩ {1, 2, 3, 4, 7}
Therefore, P' ∩ Q' = {1, 2, 3, 7} ……………….. (ii)

Combining (i) and (ii) we get;


(P ∪ Q)' = P' ∩ Q'. Proved

EXAMPLES
( A  B)'  A' B' and ( A  B)'  A' B'
We can verify de Morgan’s laws by showing that the LHS = RHS of the De Morgan’s law.

Example
Let 𝑈 = {0, 1, 2, 3, 4, 5, 6, 7, 8, 9, 10} and = {𝑠𝑒𝑡 𝑜𝑓 𝑒𝑣𝑒𝑛 𝑛𝑢𝑚𝑏𝑒𝑟𝑠 𝑙𝑒𝑠𝑠 𝑡ℎ𝑎𝑛 10} , and 𝐵 =
{𝑠𝑒𝑡 𝑜𝑓 𝑝𝑟𝑖𝑚𝑒 𝑛𝑢𝑚𝑏𝑒𝑟𝑠 𝑙𝑒𝑠𝑠 𝑡ℎ𝑎𝑛 9}. Verify the De Morgan’s laws:
(i) ( A  B)'  A' B'
(ii) ( A  B)'  A' B'

Page 11 | H A M A C H I L A . I
Solutions
 You find list the elements of A and B first as; A  0,2,4,6,8 , B  2,3,5,7 and we then find

A  B  2 and we find the complement of this intersection ( A  B)'  0,1,3,4,5,6,7,8,9,10 .


Then you find A complement and B complement as: A' 1,3,5,7,9,10 and B' 0,1, ,4,6,8,9,10 , then

we find A'B ' )  0,1,3,4,5,6,7,8,9,10

Now you can see that ( A  B)'  A' B' = 0,1,3,4,5,6,7,8,9,10


Hence, De Morgan’s law has been verified.
 You should try ( A  B)'  A' B'
TIP: De Morgan’s laws are very important, always examinable. You must know them by heart, both
stating, proving and applications.

Activity
The Venn diagram shows the elements of three sets P, Q and R in the universal set E.

List the elements of each of the following sets:

(i) P (ii) P’ (iii) Q ∩ R (iv) P \ R (v) R – Q (vi) (P ∪ R) \ Q

(vii) (P ∩ R) \ Q (viii) (P ∪ Q ∪ R)’

Page 12 | H A M A C H I L A . I
SETS OF NUMBERS
Notations and symbols
N  1,2,3,4,....., the set of natural numbers or counting numbers
W  0,1,2,3,4,5,......., the set of whole numbers
Z  ....,3,2,1,0,1,2,3,4,...... , the set of integers. Note that the set of integers has three sets of numbers,
namely the negative numbers, zero and positive numbers.
What this means is the this set contains the set of whole numbers W, and the set of Natural numbers N. in

symbols we write, N  W  Z . The set of positive integers can be written as: Z  1,2,3,4,5,.....and

the set of negative integers can be written as: Z  ........  3,2,1, the two set plus 0 gives the set of

integers.
The next set to this set of integers is called the set of rational numbers.

a 
Definition 1: A rational number is defined as ; Q   , b  0, where a, b  R  , that is the set of rational
b 
a
numbers can be written in the form , b  0 where a and b are integers. Note that in mathematics division
b
by 0 is not allowed, therefore, where an unknown appears in the denominator, it is only valid in a case
a
where it assume that the value is not zero in the denominator. A number in the form can be expressed
b
1 3 2
in decimal form, and two possibilities arises: (1) either the decimal terminates such as: , , , etc.
2 4 5
1
These have terminal decimals. (2) Repeated or non-terminal decimals such as:  0.3333........ ,
3
22
 3.1428571428 57.....
7
This set of rational numbers contains all integers, in symbols we write: Z  Q , since whole numbers can

a 3
be written in the form , for example, 3  , hence it is a rational number by definition.
b 1
Definition 2: A rational number can be written as a terminating decimal or decimals repeat. For example
3 1
 0.75 , this is an example of terminating decimals,  0.33333 ....... , this is an example of repeating
4 3

Page 13 | H A M A C H I L A . I

1
decimals. This type of decimals can be expressed in another shorter form, that is,  0.3333......  0.3 ,
3
the bar on top shows that 3 repeats.
3
What is known is that o.75 can easily be converted to a fraction which is . What would be interesting to
4

here is how 0.3333………. can be converted to a fraction. Let a  0. 3 , multiply 10 both side of the
equation. We multiply by 10 because the repeating decimal has on digit after the decimal point. If the
repeating digits were two we would have multiplied by 100.

10a  3. 3 ……(1)

a  0. 3 …….(2)
Subtract (2) from (1)  9a  3 , Divide by 9 both sides

3 1
a  , a 
9 3
___ _
Other examples are: 2.412 means 2.412121212..... , that is 12 repeats, 1.5437 =1.54377777....
a
If a number cannot be written in the form , b  0 where a and b are integers or cannot be written as
b
terminating decimals or repeated decimals, then this number is an Irrational number(I). That is,
Q  I   , but Q  I  R ,that is the set of rational numbers plus the set of irrational numbers equal the
set of Real numbers. That is the set of real numbers contains rational numbers and irrational numbers. That
is, a number is either a rational number or an irrational number and not both. In the following example we
use this principle which states that if a number is not rational, then it must be irrational.

Example 1.4.1 Prove that 2 is irrational.


a
Proof : We assume that 2 is rational. Since it is rational, then we can write it in the form , b  0 . Note
b
that this assumption is what will help us to conclude as to whether the number is rational or irrational.
a a
That is 2 , b  0 , Take the equation 2  , equation (1), where a and b are prime ( that
b b
mean a and b have no common factor). And square both sides, and we get

a2
2 2
, 2b  a ,
2 2
(2)
b

Page 14 | H A M A C H I L A . I
This implies a is divisible by 2, then a is divisible by 2. That is a has a factor 2 say a  2r , for some
2

integer r. We substitute a = 2r in (2), and we get;

2b2  (2r )2 , this implies, 2b 2  4r 2 , b 2  2r 2 , this implies b 2 is divisible by 2, then b is divisible by 2.


Now we have found a common factor 2 this is a contradiction by (1)

Therefore, 2 is not rational, hence 2 is irrational.


Note that the set of real numbers is the union of the set of rational number and the set of irrational number.
This implies that R  Q  I , a real number is either rational or irrational, that is Q  I   . The set of real
numbers are closed on the number line.
Now that you have followed well the whole procedure, try to do the next example before checking at what
we have done for you. This will help you to consolidate the understanding of the concept.

Example 1.4.2 Prove that 3 is not rational?


a
Proof: Assume that 3 is rational. If it is rational, then it can be expressed in the form were a, b are
b
integers, and prime (meaning they have no common factors). But any integer can be written in the form

 3n  9n 2
 
3n   3n  1 for n  Z  9n 2   9n 2  6n  1 equation (1)
3n  2 9n 2  12n  4
 

Therefore, we can now write 3 as a rational number by the assumption.


a
3 , where b  0 , and a and b are integers. We square both sides, and
b
obtain,

a2
3
b2 equation (2)
3b 2  a 2
2
Therefore, a is a divisible by 3, then by (1) a is divisible by 3, this implies a has a factor 3. We equate

a  3n and substitute it in equation (2). Then


3b 2  (3n) 2
3b 2  9n 2
b 2  3n 2
2
This implies, b is a divisible by 3, then by (1) above b is divisible by 3, this implies b has a factor 3.

Page 15 | H A M A C H I L A . I
This is a contradiction since our assumption is that a and b are prime, but we have found a common factor

3. Therefore, our assumption is false, that is 3 , is not rational.


Hence, 3 is irrational.

Example 1.4.2
Find an irrational number between the interval (a, b).
Solution. Note that a and b can be two real numbers such that b > a. If this condition is not satisfied, the
method demonstrated here may not work. Begin by finding the interval length of this interval. And assume

that we know that 2 is an irrational number. (which can be proved as in example 1.4.1).

b  a  0 , we divide this 2 and we have.


ba
0  b  a Here we form an inequality, then we add a throughout and the inequality
2
is maintained.
ba
0a   a b  a  a
2
ba
a ab
2
ba
We know that is an irrational number
2
ba
Therefore the required irrational number is a
2
ADDITIONAL QUESTIONS AND SOLUTIONS
Question 1.
a) Prove that √10 is irrational.

Solution: assume 10 is rational for a contradiction. Then there are a ∈ Z, b ∈ N such that
and a,b have no factors in common.
Squaring both sides and rearranging gives 10b2 = a2. Therefore both 5 and 2 are factors of a2, and hence of
a (using the argument given in lectures). Using the Fundamental Theorem of Arithmetic, there is an
integer n such that a = 10n. Putting this into 10b2 = a2 gives (on cancelling factors of 10) b2 = 10n2.
Therefore b has a factor of 10 (because b2 does), so a and b have a factor in common. This is a
contradiction, and so√ 10 is irrational.

Page 16 | H A M A C H I L A . I
b) Hence prove that 2+ 5 is irrational. (Hint: Assume by contradiction that , square the
equation and then try to simplify first.) Solution: following the hint, we get

,
from which it follows that

.
The right-hand side of this equation is a rational number, implying that
√10 ∈√Q. But this contradicts part (a), from which we conclude that √2 + √5 is irrational.

REAL NUMBERS
You can represent a subsets of real numbers in interval form such as: (𝑎, 𝑏) is an open interval, [𝑎, 𝑏] as a
closed interval, (𝑎, 𝑏] as an open-closed interval, and [𝑎, 𝑏) as a closed- open interval. In lower mathematics
these sets where represented using set builder notation for instance (a, b)  x : a  x  b, x  R , while

[a, b]  x : a  x  b, x  R, and these can be represented on a number line as illustrated below.
Example 1.4.2 Given that the universal set is R, A  (1,4] and B  [0,7] . Find the following sets and

illustrate them on the number line. (i) A' (ii) A B (iii) A


' B
Solutions
(i) You draw a number line to help you visualize the given set. Note that the interval
(-1, 4] is open-closed interval and this can be illustrated on a number line as follows:

Therefore A'  (,1]  (4, )


(ii) The shaded part in the number line is the intersection of A and B as shown below:

Page 17 | H A M A C H I L A . I
That is A  B  [0,4]

1.4.1 SOME PROPERTIES OF REAL NUMBERS


1. Identity element.
0 is an identity element under the operation +; 0+4=4
1 is an identity element under the operation x, 1x4=4

2. commutative properties
The operation + is commutative; 𝑎 + 𝑏 = 𝑏 + 𝑎, for real numbers 𝑎 𝑎𝑛𝑑 𝑏
The operation 𝑥 is commutative; 𝑎𝑥𝑏 = 𝑏𝑥𝑎 , for real numbers 𝑎 𝑎𝑛𝑑 𝑏
3. Inverse element
The operation + has an inverse element 0, for real number 𝑎
1
The operation 𝑥 has an inverse element a , for real number 𝑎
4. Transitive property
if a  b and b  c ,then a  c
5. Associative property
The operation + is associative
The operation 𝑥 is associative
6. The distributive property
The operation 𝑥 is distributive over +: 𝑎(𝑏 + 𝑐) = 𝑎𝑏 + 𝑎𝑐

Page 18 | H A M A C H I L A . I
RADICALS
We’ll start this section by defining a radical. If 𝑛 is a positive integer that is greater than 1 and 𝑎 is a real
1

number then, n
a  a n where n is called the index, 𝑎 is called the radicand, and the symbol is called

the radical. The left side of this equation is called the radical form and the right side is called the exponent
form. The left side of this equation is often called the radical form and the right side is often called the
exponent form. From this definition we can see that a radical is simply another notation for the first rational
exponent that we looked at in the rational exponents section. Note as well that the index is required in these
to make sure that we correctly evaluate the radical. There is one exception to this rule and that is square

root. For square roots we have, 2


a  a that is for square root the 2 is dropped and you just write square
root of a.
Example 1 Write each of the following radicals in exponent form.
4
(a) 16
10
(b) 8x

(c) x2  y2

Page 19 | H A M A C H I L A . I
Solutions
1
(a) 4
16  (16) 4

(b) 10
8 x  (8 x) 10
1
(c) x2  y 2  (x2  y 2 ) 2
As seen in the last two parts of this example we need to be careful with parenthesis. When we convert to
exponent form and the radicand consists of more than one term then we need to enclose the whole radicand
1
in parenthesis as we did with these two parts. To see why this is consider the following, 8x 10 From our
discussion of exponents in the previous sections we know that only the term immediately to the left of the
1

exponent actually gets the exponent. Therefore, the radical form of this is, 8 x 10
 810 x  10 8 x
So, we once again see that parenthesis are very important in this class. Since we know how to evaluate
rational exponents we also know how to evaluate radicals.
Example 2 Evaluate each of the following.

(a) 16 and 4 16
5
(b) 243
(c) 3
 125
4
(d) 16
Solution
1 1 1 1

(a) 16  (16)  (4 )  4 and 16  (16)  (2 )  2


2 2 2 4 4 4 4

1 1

(b) 5
243  (243) 5  (35 ) 5  3
1 1

(c) 3
 125  (125)  ((5) )  5
3 3 3

(d) 4
 16  (16) This cannot be evaluated since there is no real number which be found under the fourth
4

root of -16.
Note however that we can evaluate the radical of a negative number if the index is odd as the previous part
shows.
We can also write the general rational exponent in terms of radicals as follows.

Page 20 | H A M A C H I L A . I
m 1 m 1

a  (a )  a
n m n n m
or a n
 (a ) m  (n a ) m
n

m 1 m 1

a n  (a n ) m  ( n a ) m or a n  (a m ) n  n a m
Properties
If n is a positive integer greater than 1 and both a and b are positive real numbers then,

(a) n
an  a

(b) n
ab  n a n b
a na
(c) n 
b nb
Note that on occasion we can allow a or b to be negative and still have these properties work. When we
run across those situations we will acknowledge them. However, for the remainder of this section we will
assume that a and b must be positive.
Also note that while we can “break up” products and quotients under a radical we can’t do the same thing
for sums or differences. In other words,

For example ab  a  b


If we “break up” the root into the sum of the two pieces we clearly get different answers! So, be careful to
not make this very common mistake!
We are going to be simplifying radicals shortly so we should next define simplified radical form. A radical
is said to be in simplified radical form (or just simplified form) if each of the following are true.
1. All exponents in the radicand must be less than the index.
2. Any exponents in the radicand can have no factors in common with the index.
3. No fractions appear under a radical.
4. No radicals appear in the denominator of a fraction.

Example 3 Simplify each of the following. Assume that 𝑥, 𝑦, 𝑎𝑛𝑑 𝑧 are positive.

(a)

(b)

(c)

(d)

Page 21 | H A M A C H I L A . I
(e)

(f)

Solution
(a) In this case the exponent (7) is larger than the index (2) and so the first rule for simplification is violated. To
fix this we will use the first and second properties of radicals above. So, let’s note that we can write the
radicand as follows.

y7  y6 y  ( y3 )2 y
So, we’ve got the radicand written as a perfect square times a term whose exponent is smaller than the

index. The radical then becomes, y 7  ( y 3 ) 2 y . Now use the second property of radicals to break up

the radical and then use the first property of radicals on the first term. y7  ( y3 )2 y  y3 y
This now satisfies the rules for simplification and so we are done.
How do we figure out how to break up the exponent as we did? To do this we noted that the index was
2. We then determined the largest multiple of 2 that is less than 7, the exponent on the radicand. This is
6. Next, we noticed that 7=6+1.

(b) This radical violates the second simplification rule since both the index and the exponent have a
common factor of 3. To fix this all we need to do is convert the radical to exponent form do some
simplification and then convert back to radical form.
1 6 2 1
9
x  (x )  x  x  (x )  3 x 2
6 6 9 9 3 2 3

(c) There is more than one term here but everything works in exactly the same fashion. We will break
the radicand up into perfect squares times terms whose exponents are less than 2 (i.e. 1).

18x 6 y11  9 x 6 y10 (2 y)  9( x 3 ) 2 ( y 5 ) 2 (2 y) . Don’t forget to look for perfect squares in the number as
well. Now, go back to the radical and then use the second and first property of radicals as we did in the first
example.

18 x 6 y 11  9( x 3 ) 2 ( y 5 ) 2 (2 y )  9 ( x 3 ) 2 2 y  3 x 3 y 5 2 y
Note that we used the fact that the second property can be expanded out to as many terms as we have in the
product under the radical. Also, don’t get excited that there are no x’s under the radical in the final
answer. This will happen occasionally.

Page 22 | H A M A C H I L A . I
(d) This one is similar to the previous part except the index is now a 4. So, instead of get perfect
squares we want powers of 4. This time we will combine the work in the previous part into one step.
4
32 x 9 y 5 z 12  16 x 8 y 4 z 12 (2 xy)  4 16 4 ( x 2 ) 4 4 y 4 4 ( z 3 ) 4 4 2 xy  2 x 2 yz 3 4 2 xy
(e) Again this one is similar to the previous two parts.
5
x 12 y 4 z 24  5 x 10 z 20 ( x 2 y 4 z 4 )  5 ( x 2 ) 5 5
(z 4 )5 5
x2 y4 z4  x2 z4 5 x2 y4 z4
(f) Individually both of the radicals are in simplified form. However, there is often an unspoken rule
for simplification. The unspoken rule is that we should have as few radicals in the problem as possible. In
this case that means that we can use the second property of radicals to combine the two radicals into one
radical and then we’ll see if there is any simplification that needs to be done.
3
9x 2 3
6 x 2  3 (9 x 2 )(6 x 2 )  3 54 x 4
Now that it’s in this form we can do some simplification.
3
9x 2 3
6 x 2  3 27 x 3 (2 x)  3 27 x 3 (2 x)  3 x 3
2x
If we are looking at the product of two radicals with the same index then all we need to do is use the second
property of radicals to combine them then simplify. What we need to look at now are problems like the
following set of examples.

Example 4 Multiply each of the following. Assume that 𝑥 is positive.

(a) ( x  2)( x  5)

(b) 
(3 x  y ) 2 x  5 y 
(c) (5 x  2)(5 x  2)
Solutions

(a) ( x  2)( x  5)  x  5 x  2 x  10 y

= x  3 x  10

(b) (3 x   
y ) 2 x  5 y  6 x  15 xy  2 xy  5 y

= 6 x  17 xy  5 y

(c) (5 x  2)(5 x  2)  25x  10 x  10 x  4

= 25x  4
We look at some simplification examples illustrating the final two rules. Note as well that the fourth rule
says that we shouldn’t have any radicals in the denominator. To get rid of them we will use some of the

Page 23 | H A M A C H I L A . I
multiplication ideas that we looked at above and the process of getting rid of the radicals in the denominator
is called rationalizing the denominator. In fact, that is really what this next set of examples is about. They
are really more examples of rationalizing the denominator rather than simplification examples.
Example 5 Rationalize the denominator for each of the following. Assume that x is positive.
1
(a)
x

(b) 3
x2
3
(c)
1 x
1
(d)
x 3
2
(e)
x 2
Solutions
(a)
Rationalizing the denominator may seem to have no real uses in an Algebra class now. However, as you
progress in calculus class you will find that rationalizing is useful on occasion at that level. Recall that when
we first wrote down the properties of radicals we required that ‘a’ be a positive number. This was done to
make the work in this section a little easier. However, with the first property that doesn’t necessarily need
to be the case.
Here is the property for a general a (i.e. positive or negative)

 a if n is even
n
an  
 a if n is odd

Where a is the absolute value of a. All that you need to do is know at this point is that absolute value

always makes a a positive number.


So, as a quick example this means that,
6
x6  x and
7
x7  x
For square roots this is,

x2  x

Page 24 | H A M A C H I L A . I
SURDS
Surds are numbers left in root form (√) to express its exact value. It has an infinite number of non-
recurring decimals. Therefore, surds are irrational numbers.
There are certain rules that we follow to simplify an expression involving surds. Rationalising the
denominator is one way to simplify these expressions. It is done by eliminating the surd in the
denominator. This is shown in Rules 3, 5 𝑎𝑛𝑑 6.
It can often be necessary to find the largest perfect square factor in order to simplify surds. The largest
perfect square factor is found by looking at any possible factors of the number that is being square rooted.
Lets say that you are looking at the square root of 242. Can you simplify this? Well, 2 𝑥 121 𝑖𝑠 242 𝑎𝑛𝑑
we can take the square root of 121 without leaving a surd (because we get 11). Since we cannot take the
square root of a larger number that can be multiplied by another to give 242 then we say that 121 is the
largest perfect square factor.

Six Rules of Surds

Rule 1:

An Example:
Simplify :
Since , as 9 is the largest perfect square factor of 18.

Rule 2:

An Example:

Simplify :

Page 25 | H A M A C H I L A . I
Rule 3:

By multiplying both the numerator and denominator by the denominator you can rationalise the
denominator.
An Example:

rationalise :

Rule 4:

An Example:
Simplify :

Rule 5:

Following this rule enables you to rationalise the denominator.


An Example:

Rationalise :

Page 26 | H A M A C H I L A . I
Rule 6:

Following this rule enables you to rationalise the denominator.


An Example:

Rationalise :

You have now learnt the important rules of surds.


TIP: These 6 rules of surds if well understood, you can never fail any question on surds. There has never
been an exam without surds. This is the easiest way one can help you understand surds for they are very
important in mathematics.

Page 27 | H A M A C H I L A . I
COMPLEX NUMBERS
In the radicals section we noted that we won’t get a real number out of a square root of a negative

number. For instance  4 isn’t a real number since there is no real number that we can square and get
a negative 4. But we have some imaginations of what the answer may look like, we know we should have
an answer but of course not real number.

Now we also saw that if a and b were both positive then, ab  a b . For now let’s forget that

restriction and do the following.  4  (4)(1)  4  1

Now,  1 is not a real number, but if you think about it we can do this for any square root of a negative
number. For instance,

 16  (16)(1)  4  1

 7 (7)(1)  7  1
etc
So, even if the number is not a perfect square we can still always reduce the square root of a negative

number down to the square root of a positive number (which we or a calculator can deal with) times  1

So, if we just had a way to deal with  1 we could actually deal with square roots of negative

numbers. Well the reality is that, at this level, there just is not any way to deal with  1 so instead of
dealing with it we will “make it go away” so to say by using the following definition.

i  1
Note that if we square both sides of this we get, i 2  1 .
It will be important to remember this later on. This shows that, in some way, i is the only “number” that
we can square and get a negative value.

Using this definition all the square roots above become,

 25  5i
 7 7i
 49  7i

Page 28 | H A M A C H I L A . I
 81  9i etc

Definition: A complex number is a number which can be expressed in the form Z  a  ib where 𝑎 𝑎𝑛𝑑 𝑏
real numbers and 𝑎 is called the real part and 𝑖𝑏 is called the imaginary part.
So a complex number has two parts the real part and an imaginary part, hence this is a bigger set than the
set of real numbers.
Here are some examples of complex numbers:
1 2i
1
 5i
3
 3  7i
1 i 3
3  2i
These are complex numbers which has a real part and imaginary part
2 , is purely real since the imaginary part is 0, but it can be written as 2  0i
3i , is purely imaginary since the real part is 0, but it can be written as 0  3i
The set of complex numbers contains the set of real numbers, in symbols R  C , that is the set of real
numbers is a proper subset of Complex numbers.
If a complex number of the form a  bi , then its conjugate is a  bi , that is, to get a conjugate of any
complex number you simply change the sign of the imaginary part, if it was + change to – and vice-versa.
Here are some examples of complex numbers and their corresponding conjugates:
Complex number Its Conjugate
1 5i 1 5i
3i 3i
3i  3i
5 5
Notice that the conjugate of a real number is just itself with no changes.
A complex number in standard form is in the form a  bi . Therefore a purely real complex number is
written as a  0i , while a purely imaginary complex number is written in the form 0  bi .

Page 29 | H A M A C H I L A . I
The next topic that we want to discuss here is powers of i. Let’s just take a look at what happens when we
start looking at various powers of i.

Can you see the pattern? All powers if i can be reduced down to one of four possible answers and they
repeat every four powers. This can be a convenient fact to remember.
Stop and think: what is a complex number? Are all real numbers complex, are natural numbers complex,
are rational or irrational numbers complex according to the definition? Don’t forget the definition of i. lets
now move and look at operations on complex numbers.

Arithmetic of Complex Numbers


(Add, Subtract, Multiply)

Add and Subtract Complex Numbers

Page 30 | H A M A C H I L A . I
When performing the arithmetic operations of adding or subtracting on complex numbers, remember to combine "similar" terms.
Also check to see if the answer must be expressed in simplest a+ bi form.

Addition Rule: (a + bi) + (c + di) = (a + c) + (b + d)i

Add the "real" portions, and add the "imaginary" portions of the complex numbers.
Notice the distributive property at work when adding the imaginary portions.

Additive Identity: (a + bi) + (0 + 0i) = a + bi

Additive Inverse: (a + bi) + (-a - bi) = (0 +0i)

ADD: (6 + 4i) + (8 - 2i)


(6 + 4i) + ( 8 - 2i) = (6 + 8) + (4 - 2)i = 14 + 2i

ADD: Express answer in a + bi form.

ADD: Express answer in a + bi form.

Subtraction Rule: (a + bi) - (c + di) = (a - c) + (b - d)i

Subtract the "real" portions, and subtract the "imaginary" portions of the complex numbers.
Notice the distributive property at work when subtracting the imaginary portions.

SUBTRACT: (10 + 3i) - (7 - 4i)


Express answer in a + bi form.

Page 31 | H A M A C H I L A . I
(10 + 3i) - (7 - 4i) = (10 - 7) + (3 - (-4))i = 3 + 7i

SUBTRACT:
Express answer in a + bi form.

SUBTRACT:
Express answer in a + bi form.

Multiply Complex Numbers

Multiplying two complex numbers is accomplished in a manner similar to multiplying two binomials.
The distributive multiplication process (sometimes referred to as FOIL) is used.

Distributive Multiplication

Remember that
i 2 = -1

Be sure to replace i2 with (-1).

Page 32 | H A M A C H I L A . I
Multiplication Rule: The product of two complex numbers is a complex number.

The final result is expressed in a + bi form and is a complex number.

Multiplicative Identity: (a + bi) • (1 + 0i) = a + bi

Mutiplicative Inverse:
The number (0 + 0i) has no multiplicative inverse.

The conjugate of a complex number a + bi is the complex number a - bi.


For example, the conjugate of 3 + 7i is 3 - 7i.
(Notice that only the sign of the bi term is changed.)
If a complex number is multiplied by its conjugate, the result will be a positive real number
(which, of course, is still a complex number where the b in a + bi is 0).

The product of a complex number and its conjugate is a real number,


and is always positive.

This answer is a real number (no i's).


In addition, since both values are squared, the answer is positive.

Page 33 | H A M A C H I L A . I
Compute: (2 + 3i)•(1 + 5i)
(2 + 3i)• (1 + 5i) = 2(1 + 5i) + 3i(1 + 5i) = 2 + 10i + 3i + 15i2
= 2 + 13i + 15(-1) = -13 + 13i

Compute: (2 + i)2
(2 + i)• (2 + i) = 2(2 + i) + i(2 + i) = 4 + 2i + 2i + i2
= 4 + 4i + (-1) = 3 + 4i

Compute: (3 - 2i)•(1 - 4i)


(3 - 2i)• (1 - 4i) = 3(1 - 4i) + (-2i)(1 - 4i) = 3 - 12i - 2i + 8i2
= 3 - 14i + 8(-1) = -5 - 14i

Compute: (3 +4i)•(3 - 4i) (conjugates!)


(3 + 4i)• (3 - 4i) = 3(3 - 4i) + 4i(3 - 4i) = 9 - 12i + 12i - 16i2
= 9 - 16(-1) = 25 (a real number)
Division

Division is achieved through use of the complex congugate, , i.e. with the sign of the imaginary part reversed. It
will be noted that

i.e. a real, positive number.

Division of a complex number by another complex number is facilitated by multiplying both numerat

and denominator by giving

.
3.1 Examples

Convert the fraction to the form .


Solution:

Page 34 | H A M A C H I L A . I
.
Notice that we have given you a lot of examples on complex numbers, we understand it may be a bit strange for you to work with
complex numbers because of the introduction of i. Now, have you understood, do you now know what complex numbers are, can
you add, subtract, multiply and divide the complex numbers, if so you can proceed.

BINARY OPERATIONS
In mathematics we are interested in the study of basic operations of algebra, such as: , , ,  . You have
studied these operations at school, but in this section, you will study the use of these operations in details.
Definition: The operation * is a binary operation in S, if and only if, for every a, b  S , the a * b  S
Otherwise the operation * is not a binary operation in S.
A binary operation on a set A is nothing more than a function from A× A to A. In other words, it takes pairs
of values from A and converts them into single values from A.
You know the product of 3 and 4 as 12. In this case the operation is multiplication, that is *   .
This * can be defined in any way different from the four basic operations given above as will be illustrated
below. Though it may be hard to understand, there is nothing difficult about binary operations, we just
follow how they are defined and then check if the answer is in the given set of numbers.
Example 1.4.3

(a) State whether the * defined as: a * b  a  b is a binary operation in the set of real numbers.
Solution: To show that * is a binary operation or not, all what is required is to find choices of a and b were

a*b fails. If a=1 and b=5, then a * b  1  5   4 , then  4  R , therefore * is not a binary operation
in the set of real numbers.
(b) State whether * defined as *   , in the set of real numbers. Is * commutative.
a
Solution: Take any two numbers in the set of real numbers, say a, b  R , then a * b  a  b  R,
b
a b
hence *   is a binary operation on R. But *   is not commutative since a * b    b * a we also
b a
need to know how to check commutativity, it’s very simple, just check if a*b = b*a, that’s all. Are you not
happy, you thought it will be difficulty? It’s the simplest. We also need to know how to check associativity,
it’s also simple, just check if a*(b*c) = (a*b)*c. if so then * is associative but if not then * is not associative.

Page 35 | H A M A C H I L A . I
Stop and think: what is a binary operation? Can you show if the operation is commutative, associative
and if it is binary in a given set, if you can, try the following exercise.
Note that these are examination samples, they are easy especially if you do them just now and at least revise
one or two times before an examination, you will really enjoy in an examination.
Exercise 1

Exercise 1.1
1. If A={ 1,2,3,4 } ,B={ 2,4,6,8 } , C ={ 3, 4 , 5, 6 } and the universal set X ={ 1,2,3,4,5,6,7,8,9 }.

(a) Find (i) A’ (ii) ( A  B) ' (iii) B – C (iv) ( A  B) ' (v) A  ( B  C )


(vi) ( X  C )  ( A  B)
'

(b) Confirm
(i) The associative laws: ( A  B)  C  A  ( B  C ) and ( A  B)  C  A  ( B  C )

(ii) the distributive laws: A  ( B  C )  ( A)  ( A  C ) and

A  ( B  C )  ( A  B)  ( A  C )

(iii) The De Morgan’s laws: ( A  B)  A  B and ( A  B) '  A '  B '


' ' '

2. (a) If A  B , then simplify if possible (i) ( A  B) (ii) A '  B ' (iii) A  B '

(iv) A'( A  B)
3. In each problem below, one of the following relations is true:

A  B, A = B, B  A. Write the correct relation in each case.


A B
a) { x│2x + 3 = 11 – 2x } {x│5x + 4 = x + 12}
b) { x│x2 + 4 = 40 – 6x} {x│4 + 2x = 10}
c) { x│(x – 2)(x – 3) = 0} {x│ x = 2}
d) { x│ x + 4 = 0} {x │x(x + 4) = 0}
e) { x│ x – 1 = 0} U { x│ x – 2 = 0} { x│ x2 – 3x + 2 = 0}
f) { x │x + 3 = 4} { x│(x + 3)2 = 16}
4. Rewrite the following in set builder form:
a) A = [-3, 5) b) B = (3, 8) c) C = [0, 4) d) (-7, -2]
5. Prove that A – B = A ∩ B′
6. Express each of the following in its simplest form
a) [P′ U (Q – P)]′ b) X U (Y U X) c) ( M ∩ N) U ( M ∩ N′) d) A – ( A – B)

Page 36 | H A M A C H I L A . I
In the Venn diagram below, shade: (i) W – V (ii) V  W (iii) V  W
' '
7.
(iv) V’ – W’

a
8. In problems (a) to (h) find expression of the form for the given decimal expansions, where a and b are
b
integers.

(a) 0.5 (b) 3.12 (c) 11.34 (d)  4.357 (e) 9.413
(f) 0.9999… (g)1.3333…. (h) 0.85
9. (a) Find an irrational number which lie between (i) 2 and 3 (ii) 19 and 19.01
(iii) -4 and -2
(b) Each of the following operation in I,II and III is a binary operation on R.
I: a*b = (a – b)(a + b) , a, b  R,
II: a*b = ab , a, b  R,
III: a*b = 2 a b , a, b  R
(i) Determine which of I, II III is commutative
(ii) For each of I , II and III , evaluate (3* 2 ) * 5
10. Given the sets X = { 0, 1 } and Y = { 0, 1, 2 }
(a) Check whether each of the following operations +, -, x , ÷ is a binary operation on X and Y.
(b) Also check whether the operation is commutative and associative
11. If A = [ 1, 4] , B = ( 2, 8 ) , C = [ 3, 6 ) and the universal set X = [1, 9 ] . Find each of the following sets

and display it on the number line. (i) B’ (ii) ( A  B) (iii) B – C (IV) ( A  B) (v) A  ( B  C )
' '

(vi) ( X  C )  ( A  B) .
'

12. An operation * is defined on the set { 3,5,7 } in the table below as follows: a*b is the result where the
row along “a” and column a long “b” meet. e.g. 5*7 = 7.

Page 37 | H A M A C H I L A . I
(i) Is this operation a binary operation on set { 3,5,7 }
(ii) Is this operation commutative
(iii) Evaluate (5*7)*3

MATRICES

Introduction
We begins by reviewing basic operations of matrices; these are addition, subtraction, and multiplications
of matrices. The ‘O ‘level knowledge of matrices is extended from a 2x2 matrices to a 3x3 matrices. The
chapter discusses determinants, transpose, and inverses of a 3x3 matrices. Lastly, we explore the methods
of solving systems of equations using matrices.

Operations on matrices
A matrix is an array of numbers arranged in rows and columns, such as

 3 4  2 2
2 3    0 
0 1 ,  1 0 3  or  
   2 1 2   3

Such an array of numbers is called a Matrix. The numbers such as; 2, 3 for first one are called the elements
or entries of the matrix. The horizontal line 2, 3 is called first row while the vertical line i 2 0 is called first
column. In general, if a rectangular array has m rows and n columns, you call it m x n matrix. Therefore,
the first matrix given above is a 2x2 matrix, it means it has two rows and two columns. The number of rows
is stated first, followed by the number of columns. Then, the second matrix is a 3x3 matrix and the last
matrix is a 3x1 matrix meaning it has 3 rows and 1 column.

Basic properties of Matrices


You use capital letters to denote matrices, and you enclose actual matrix in square brackets.
Definition: Two matrices 𝐴 𝑎𝑛𝑑 𝐵 are equal if and only if:
1. The two matrices have the same number of rows and columns
2. Their corresponding elements are equal.
Definition: The sum of two matrices 𝐴 𝑎𝑛𝑑 𝐵 denoted by 𝐴 + 𝐵, is the matrix such that each of its
elements is the sum of the corresponding elements of 𝐴 𝑎𝑛𝑑 𝐵.

Page 38 | H A M A C H I L A . I
2 3 1 2
Example If A    , B  , find 𝐴 + 𝐵.
1 5 0 4 
2  1 3  2 3 5
Then A  B     with the definition of any operation, you can now think of the
1  0 5  4 1 9
possibility of an “identity matrix” of the operation addition.
Definition: If all of the entries of a matrix are zeros, the matrix is called the zero matrix, and is denoted by
O.

0 0 
0 0 That is 𝐴 + 𝑂 = 𝐴 for any matrix A. Furthermore, there is the inverse matrix with respect
 
to addition called the negative of 𝐴 and denoted by – 𝐴, such that 𝐴 + (−𝐴) = 𝑂. Note that you can only
add matrices of the same size or matrices of the same order.

Definition: The product of a scalar 𝑘 and a matrix 𝐴, denoted by 𝑘𝐴, is the matrix in which each entry is

2 3 4 6 
𝑘 times the corresponding element of 𝐴. For example, if A    , then 2A    . Where 𝑘 = 2.
1 5 2 10
Definition: The product of two matrices A and B is the matrix AB whose entry in the ith row and jth column
is the sum of the products formed by multiplying each entry in the i th row of A by the corresponding entry
in the jth column of B.

0 2 
3 2  1
Example: If A    , and B  1 3  , find AB and BA
 
1 2 4  5  2

Solution: You can multiply matrices if they are of the same order, or the number of elements of the rows
of the first matrix is equal to the number of elements of the columns of the second matrix. For instance, a
2x3 matrix can be multiplied by a 3x3 matrix and the product will be a 2x3 matrix. In this example you are
multiplying a 2x3 matrix by a 3x2 matrix and the product will be a 2x2 matrix.

Note that matrices are not commutative. AB  BA

Page 39 | H A M A C H I L A . I
Determinant of a matrix
You may have learnt how to find a determinant of a 2𝑥2 matrix a ‘O ‘level mathematics. In this section you
will learn how to find the determinant of a 3x3 matrix. Note that determinant can only be found in a square
matrix. You cannot find a determinant for a rectangular matrix.

a b  a b
If a matrix A=   be a 2𝑥2 matrix, the determinant of A, written det A  A   ad  bc
c d  c d
To find the determinant of a 3x3 matrix is not as easy as this one. Take for example a general 3x3 matrix

a b c a b c
  e f d f d e
like B   d e f  , then B  d e f a b c
g h i g i g h
 h i  g h i
The 3x3 matrix is decomposed in determinants of 2x2 matrix which you can easily evaluate each of the 2x2
matrices and add them to find the determinant of a 3x3 matrix.

 1 2 4
 
Example: Given that A    1 3 0  , find A .
 0 1 5
 
1 2 4
3 0 1 0 1 3
A  1 3 0 1 2 4  1(15  0)  2(5  0)  4(1  0)
1 5 0 5 0 1
0 1 5
Solution:  15  10  4
 21

That means the determinant of 𝐴 𝑖𝑠 21.

Transpose of a Matrix
If you have a matrix and take its first row and write this as the first column, then take its second row and
write this as the second column, and so no, the resulting matrix is called the transpose of the first matrix.

 2 1 2 2 1 1 
   
If A   1 4 6  , then the transpose of A, written A   1 4  1
T

 1 1 2 2 6 2 
   

The inverse of a matrix


Just as you were able to find inverse of functions, you learnt that a function has an inverse if and only if the
function is 1-1. Also here, the inverse of matrix exists if and only if the determinant of the matrix is non-

Page 40 | H A M A C H I L A . I
zero. Note that you can only find an inverse of a square matrix. You will write the inverse of a matrix A
1
as” A " .
Definition: An inverse of matrix A is defined as:
1
A1  adjo int . AT .
A
Read ad joint A matrix transpose divided by the determinant of A. What you need to learn is how to find
this ad joint matrix which has to be transposed. This ad joint matrix is a matrix of co-factors called minors
which will be demonstrated here on how to get it. A general approach will be used here on how to get the
ad joint matrix.
Take for instance the matrix

a b c
 
A  d e f
g i 
 h

The minor of this matrix are: a11 reads the element of the first row and first column, a12 , reads the element

of the first row and second column, a21 , reads element of the second row and first column, and so on.
Therefore, the minors are computed as follows:
e f d f d e
a11   ei  fh a12   di  fg a13   df  eg
h i g i g f

b c a c a b
a21   bi  ch a22   ai  cg a23   ah  bg
h i g i g h

b c a c a b
a31   bf  ce a32   af  cd a33   ae  bd
e f d f d e

Now you impose negative positive to each of these minors stating from a11 as positive and write them in
as matrix as shown below:

 (ei  fh )  (ai  cg ) (bi  ch) 


 
AdJ . A    (bi  ch) (ai  cg )  (ah  bg )  This is the ad joint matrix A
 (bf  ce)  (af  cd ) (ae  bd ) 
 

 (ei  fh )  (bi  ch) (bf  ce) 


 
Ad jo int AT    (ai  cg ) (ai  cg )  (af  cd ) 
 (bi  ch)  (ah  bg ) (ae  bd ) 
 
Therefore,

Page 41 | H A M A C H I L A . I
 (ei  fh)  (bi  ch) (bf  ce) 
11 1  
A  T
Adjo int A =   (ai  cg ) (ai  cg )  (af  cd ) 
A A 
 (bi  ch)  (ah  bg ) (ae  bd ) 

 1 2 4
 
Example: Let A    1 3 0  , Find the inverse of 𝐴.
 0 1 5
 
Solution: You find the determinant of the matrix 𝐴.
1 2 4
3 0 1 0 1 3
A  1 3 0  1 2 4  15  10  4  21
1 5 0 5 0 1
0 1 5

A  21

3 0 1 0 1 3
a11   15 a12  5 a13   1
1 5 0 5 0 1

2 4 1 4 1 2
a21   10  4  6 a22  5 a23   1
1 5 0 5 0 1

2 4 1 4 1 2
a31   12 a32   4 a33   3 2  5
3 0 1 0 1 3

 15  6  12 
 
AdjA   5
T
5 4 
 1 1 5 

 15  6  12 
1  
Therefore, A1  5 5 4 
21 
 1 1 5 

Note that A1 A  I

Simultaneous equations
You came across simultaneous equations in two equations in two unknown in junior mathematics and
solved these equations using; elimination method, substitution method, and graphical methods. In this
section you will learn how to solve systems of equations in three equations in three unknown using matrix
methods. Two methods will be demonstrated here;

Page 42 | H A M A C H I L A . I
Inverse Method
In this method you will use the fact that A1 A  I in solving system of equations. To do that you need to
express the system of equation in the matrix form like AY  B . Where A is the matrix of the coefficients
of the variables, Y is the column vector of the variables and B the column vector of constants.
Given a system of equation as:
ax  by  cz  m
dx  ey  fz  n
gx  hy  jz  p
You can express this system of equation in matrix form as;
AX  D Where the matrix A is
a b c  x m
     
A  d e f  , X   y  , and D   n  and your system of equation can be written as:
g j  z  p
 h    

a b c  x   m 
    
d e f  y    n  . You can solve this system of equation using either Inverse of a matrix or
g j  z   p 
 h
using crammers’ rule.

Inverse of a matrix method


You need to find the determinant of the matrix A, and get its inverse A1 , and using the relationship
A1 A  I , you will be able to find the values of x, y, and z.
Example: Solve the following system of equations using inverse of a matrix method.
2x  y  z  1
x  3z  2
 3 x  y  z  1
Solution: You write this system in matrix form as: AY  B

 2 1  1 x   1 
    
 1 0 3  y    2  (1)
  3  1 1  z    1
    

 2 1  1  x 1
     
Where A   1 0 3  , X   y  , and D   2 
  3 1 1  z   1
     

Page 43 | H A M A C H I L A . I
There A  3 , you can verify this value.

You now find the ad joint matrix of co factor minors:

0 3 1 3 1 0
a11  3 a12   10 a13   1
1 1 3 1  3 1

1 1 2 1 2 1
a21  0 a22   1 a23  1
1 1 3 1  3 1

1 1 2 1 2 1
a31  3 a32  7 a33   1
0 3 1 3 1 0
Therefore, the ad joint matrix transpose is;

 3 0 3   3 0 3 
  1 1  
Adj. A    10  1  7  , therefore, A 
T
  10  1  7  (2)
 1 1 1  3 
   1 1 1 
Multiply (2) to (1)

 3 0 3  2 1  1 x   3 0 3  1 
1     1   
  10  1  7  1 0 3  y     10  1  7  2 
3     3   1  1  1   1
  1  1  1   3  1 1  z    
The left hand side of the equation is A1 A  I

 1 0 0  x   3 0 3  1 
   1   
 0 1 0  y     10  1  7  2 
 0 0 1  z   3   1  1  1   1
     

 1 0 0  x   0 
   1  
 0 1 0  y     5
 0 0 1  z   3   2 
    

 x  0 
  1   5 2
 y    5  This implies, x  0, y  , z 
 z   3   2 3 3
   

Using Cramer’s rule


You can solve the same equation using crammer’s rule, where you use determinants only.

Page 44 | H A M A C H I L A . I
To use Cramer’s Rule to solve a system of three equations with three unknowns, we need to follow these
steps:
Step 1: Find the determinant, D, by using the x, y, and z values from the problem.
Find the determinant, Dx, by replacing the x-values in the first column with
Step 2:
the values after the equal sign leaving the y and z columns unchanged.
Find the determinant, Dy, by replacing the y-values in the second column
Step 3:
with the values after the equal sign leaving the x and z columns unchanged.
Find the determinant, Dz, by replacing the z-values in the third column with
Step 4:
the values after the equal sign leaving the x and y columns unchanged.
Step 5: Use Cramer’s Rule to find the values of x, y, and z.

a b c  x   m  a b c
      
Take the general form  d e f  y    n  , in here take A  d e f  and A , define the
g j  z   p  g j 
 h  h
following determinants as follows:

m b c a m c a b m
A1  n e f , A2  d n f , and A1  d e n That is you replace the ith column by D. Then
p h j g p j g h p
your x, y, and z values are found by:
A1 A2 A3
x , y , and z 
A A A

Example: Solve the following system of equations using inverse of a matrix method.
2x  y  z  1
x  3z  2
 3 x  y  z  1

 2 1  1 x   1   2 1  1
      
Solution: This system is of the form  1 0 3  y    2  and A   1 0 3
  3  1 1  z    1   3 1 1 
      

We know A  3 and you find the determinants of the Ai .

Page 45 | H A M A C H I L A . I
1 1 1 2 1 1 2 1 1
A1  2 0 3 0 A2  1 2 3  5 A3  1 0 2  2
1 1 1  3 1 1  3 1 1
Therefore, using Cremer’s rule the solutions are:
A1 0 A 5 5 A 2 2
x  0, y  2   , and z  3  
A 3 A 3 3 A 3 3

ELEMENTARY MATRIX OPERATIONS (GAUSS-JORDAN METHOD)


Elementary matrix operations play an important role in many matrix algebra applications, such as finding
the inverse of a matrix and solving simultaneous linear equations.
Elementary operations
These are three kinds of elementary matrix operations
 Interchange two rows (or columns)
 Multiply each element in a row (or column) by a none-zero number.
 Multiply a row (or column) by a non-zero number and add the result to another row (or column)
When these operations are performed on rows, they are called elementary row operations; and when they
are performed on columns, they are called elementary column operations.
Example
Solve the simultaneous equation using Gauss-Jordan elimination method.
5𝑥 + 2𝑦 = 3
−3𝑥 + 3𝑦 = 15
Solution:
A method of solving a linear system of equation. This is done by transforming the system’s augmented
matrix into reduced row-echelon form by means of row operations.

5 2 3
( ) Row operations can be used to express the matrix in reduced row-echelon form.
−3 3 15
1 1
We multiply the first row by and the second by
5 3

1 2⁄5 3⁄5
( ) We add the second row with the first
−1 1 5

1 2⁄5 3⁄5 5
( ) We multiply the second row by 7
7
0 ⁄5 28⁄5

Page 46 | H A M A C H I L A . I
1 2⁄5 3⁄5 −2
( ) We add the first row with the second one multiplied by 5
0 1 4

1 0 −1
( ) The matrix we have obtained represents the system 𝑥 = −1 𝑎𝑛𝑑 𝑦 = 4
0 1 4

Example: solve the system of equations using Gauss-Jordan method


5𝑥 + 2𝑦 =2
2𝑥 + 𝑦 − 𝑧 = 0
2𝑥 + 3𝑦 − 𝑧 = 3
Solution: The augmented matrix of the system is;
1 1 2⁄ 0 2⁄
5 2 0 2 𝑅 1 2⁄ 2⁄ 5 5
5 1 5 0 5 𝑅2 + (−2𝑅1 ) 1⁄ −4⁄
(2 1 −1 0) (2 1 −1 0 ) 0 5 −1 5
2 3 −1 3 2 3 −1 3 𝑅3 + (−2𝑅1 ) 11⁄ −1 11⁄
0 5 5)
(
1 2⁄5 0 2⁄ 1 2⁄5 0 2⁄ −1
5𝑅2 5 𝑅2 + (−𝑅3 ) 5 𝑅
(0 1 −5 −4 ) (0 −10 0 −15) 10 2
5𝑅3 1
0 11 −5 11 0 11 −5 11 𝑅
11 3
−2
2⁄ 2⁄ 𝑅1 + 𝑅 −1⁄ −11
1 5 0 5 5 2 1 0 0 5 𝑅
(0 1 0 3⁄ ) 0 1 0 3⁄ 5 3
2 𝑅3 + (−𝑅2 ) −5⁄ 2
0 1 −5⁄11 0 0 11 −1⁄
1 ( 2)
−1⁄
1 0 0 5
3⁄ −1 3 11
0 1 0 2 Therefore, 𝑥 = 5 , 𝑦 = 2
, 𝑎𝑛𝑑 𝑧 = 10
0 0 1 11⁄
( 10)

Example: Find the inverse of the matrix using Gauss-Jordan elimination method, given that
3 0 2
𝐴 = (2 0 −2)
0 1 1

Page 47 | H A M A C H I L A . I
Solution:
We start with the matrix A, and write it down with an identity matrix I next to it:
3 0 2 1 0 0 5 0 0 1 1 0 1
𝑅2 + 𝑅1 𝑅
(2 0 −2 0 1 0) (2 0 −2 0 1 0) 5 1
0 1 1 0 0 1 0 1 1 0 0 1

−1
1 0 0 0.2 0.2 0 𝑅 − 2𝑅 1 0 0 0.2 0.2 0 𝑅
(2 0 −2 0 1 0)
2 1
(0 0 −2 −0.4 0.6 0) 2 2
0 1 1 0 0 1 0 1 1 0 0 1

1 0 0 0.2 0.2 0 1 0 0 0.2 0.2 0 𝑅2 − 𝑅3


𝑅2 ⇆ 𝑅3
(0 0 1 0.2 −0.3 0) (0 1 1 0 0 1)
0 1 1 0 0 1 0 0 1 0.2 −0.3 0

1 0 0 0.2 0.2 0 0.2 0.2 0


(0 1 0 −0.2 0.3 1) Therefore the 𝐴−1 = (−0.2 0.3 1)
0 0 1 0.2 −0.3 0 0.2 −0.3 0

Exercise
1. Evaluate

 7 6  9 2  8  7  9 3
(a)      (b)     
  5 4  1  3  5 4   4  9
 1 3 2
3  5 4  
(c)     1 0 4
0  6 3  0 0  2 

2. Find the inverse of

2 1  5  2 2 1
 3 2    
(a)   (b) 1 0  2 (c)  4 1 5
 1 7 0 0 3   1 1 7
  
2  1 3  1 2 4
   
3. Given that A  1 4  7 and B   3 1  2
0 6  3   1 1 3 
 
Find (a) AT (b) BT
Hence find (c) (AB)T (d) (BA)T
4. Solve the system of equations.

Page 48 | H A M A C H I L A . I
(a) 4x + 3y + 5z = 11
9x + 4y + 15z = 13
12x + 10y – 3z = 4

(b) 5x + 4y + 2z = 16
7x – 8y + 3z = – 45
x + 6y – 4z = 16
5. Solve the system of equations
(i) Using matrix method.
(ii) Using Cramer’s rule.
(iii) Gauss-Jordan method
(a) x + y + 2z – 2 = 0 (b) x+y+z=2
– 3 x + y + 2z – 1 = 0 x + 2y + 3z = 6
6x + 2y + z + 4 = 0 3x + 2y – z = – 4

Page 49 | H A M A C H I L A . I
ALGEBRA

LINEAR AND QUADRATIC EQUATIONS

The simplest equation defined in mathematics by means of a non-trivial algebraic expression is the equation
defined by the equation 𝑦 = 𝑚𝑥 + 𝑐

Example: Solve the following equations 4 x  5  x  7


Solution:
4x  5  x  7
4x  x  7  5
3x  2
2
x
3
Example: Solve the equation x  1  3

Solution: This equation has solution x  4 . But if you square both sides you will get: ( x  1)  9 . If this
2

equation is solved, one of its roots i.e. −2, although it does not satisfy the original equation. Note that when
an expression is squared more roots are introduced in the solution set. Accepted solutions which satisfies
the original equation.
2 x  5 5x
Example: Solve the equation  x
2 x 1

Solution:
2x  5 5x
 x
2 x 1
( 2 x  5)( x  1)  5 x( 2)
x
2( x  1)
2 x2  5x  2 x  5  2 x2  2 x
x  1

The theory of quadratic equations (Maximum and Minimum)


The second type of algebraic function usually considered is one in which the defining equation is of the
second degree.

Page 50 | H A M A C H I L A . I

Definition: The function f , defined by the second degree equation f  ( x, y) : y  ax  bx  c , is
2

called a quadratic function. Note that y  f ( x)  ax2  bx  c is a quadratic function while
ax 2  bx  c  0 is called a quadratic equation. In general, the equation has solutions called zeros of a
function or the roots of the equation f ( x)  0 .
A quadratic function has a turning point which is either the maximum point or a minimum point. The

maximum or minimum point of a quadratic function can be determined by the coefficient of the x . If a  0
2

the function has a minimum, but if a  0 , the function has a maximum.


The graph of the general quadratic function may be sketched by the more direct process of expressing the

quadratic function in terms of the square of a linear function. Consider the function f ( x)  ax  bx  c ,
2

by completing squares of this function as follows: f ( x)  ax  bx  c factor out a


2

b c
f ( x)  a( x 2  x  ) , take the coefficient of x, half it and square it and add it as a zero.
a a
b b b c
f ( x)  a( x 2  x  ( )2  ( )2  )
a 2a 2a a
 b b 2
c
f ( x )  a ( x  ) 2  2  
 2a 4a a
 b 4ac  b 2 
f ( x )  a ( x  ) 2  
 2a 4a 2 

b b
Since the square quantity ( x  )  0 , the expression within the bracket has its least value when x  
2a 2a
b 4ac  b 2
. If a  0 , the function has its least value at x   . This least value of the function, ( ) , is
2a 4a
b 4ac  b 2
called its minimum. The least point, ( , ) , is called the minimum point. If, however, a  0 ,
2a 4a
b 4ac  b 2
when x   , the function has its greatest value called its maximum, and is also equal to ( ).
2a 4a
b 4ac  b 2
The greatest point, ( , ) , is called the maximum point. In either case the point,
2a 4a

Page 51 | H A M A C H I L A . I
b 4ac  b 2
( , ) , is called the vertex of the parabola, with this point found, and an additional point or
2a 4a
two points , the graph can be sketched like the one below:

Tip: maximum and minimum points are very important in sketching graphs, make sure you
understand how to get these points.

Nature of the roots of quadratic equations

The general form of a quadratic equation is: ax  bx  c  0 , where a, b & c are constants
2

The expression b  4ac is called the discriminate and it is given the letter Δ (delta).
2

All quadratic equations have two roots/solutions. These roots are real (Distinct or equal) or
complex. *complex - involving the square root of -1

You can determine the nature of the roots of a given quadratic equation by calculating the value of the

discriminant b  4ac   . If
2

(i) b 2  4ac  0 , then the quadratic equation has two distinct real roots. That is   0
(ii) b 2  4ac  0 , then the quadratic equation has one real root.

Page 52 | H A M A C H I L A . I
(iii) b 2  4ac  0 , the quadratic equation has imaginary root

These three properties are very important, always examinable and if well understood, they are the easiest
one would always want to use in any question that involves the discriminant.

The nature of the roots can be used to determine whether the solution of the given quadratic equation has
one root for (ii), two distinct roots for (i) and imaginary roots for (iii).
Example: Determine the nature of the roots of the following quadratic equations
(i) 6  x  x2  0
Solution; This equation ha a = -1, b =-1 and c = 6, the discriminant is b  4ac . On substitution
2

b 2  4ac  (1) 2  4(1)(6)  1  24  25  0


That means the equation has two distinct roots.
(ii) 2 x 2  3x  5  0
Solution; This equation ha a = 2, b =-3 and c = 5, the discriminant is b  4ac . On substitution
2

b 2  4ac  (3) 2  4(2)(5)  9  40  31  0


That means the equation has imaginary roots.

Solution of quadratic equation


You are now in a position to find the zeros of quadratic equation. Recall that the equation f ( x)  0 , which

is the values of x where y  0 , are what you are interested. Generally, two values of x can be found, but

these values can be determined by the discriminate b  4ac without solving the equation. The zeros or
2

the solutions of a quadratic equation are values of x where the graph f ( x)  ax  bx  c crosses or meets
2

the x-axis. The two roots can be found graphically, by factorization, by using a formula, or by completing
the squares. In this section a method of completing squares will be demonstrated below.

Example: Solve the following equation ax  bx  c  0 , using the method of completing squares.
2

Page 53 | H A M A C H I L A . I
Solution: Take the equation

ax 2  bx  c  0 transpose c
ax 2  bx  c Divide by a through out
b c
x 2  x   take the coefficient of x, half it and square it and add it to both sides
a a
2 2
b  b   b  c
x  x
2
   
a  2a   2a  a
2
 b  b2 c
x   
 2a  2
4a a
b 2  4ac
2
 b 
 x    take square root both sides
 2a  4a 2
b b 2  4ac
x 
2a 4a 2
b b 2  4ac
x  solve for x
2a 2a
b b 2  4ac
x  to give you the formula
2a 2a
 b  b 2  4ac
x
2a

 b  b 2  4ac  b  b 2  4ac
There are two roots or solutions such as x  or x  . These are the
2a 2a
roots, or zeros, or solutions of the quadratic equation ax  bx  c  0 in solving quadratic equation, you
2

can use any method provided the method of solving is not given in the question.

Example: Solve the equation 3x  x  10 , using completing square method.


2

Page 54 | H A M A C H I L A . I
Solution: you set
3 x 2  x  10
1 10
x2  x 
3 3
2 2
1  1 10  1
x 2
 x       
3  6 3  6
2
 1 10 1
x    
 6 3 36
120  1
2
 1
x    take square root both sides
 6 36
1 121
x   
6 36
1 11
x    solve for x
6 6
1 11
x  
6 6
5
x  2 or
3
5
The two roots are 2 or . If the method of solving is not specified in the question, any method may be used
3
and it will give the same solutions.
Example: Find the two values of x that satisfy the following quadratic equation:

In this equation, you have;

Page 55 | H A M A C H I L A . I
Relations between zeros and coefficients of the quadratic equations
If  and  are roots of a given quadratic equation, then the standard form of a quadratic equation is;

x 2  (   ) x    0 , and the general equation of a quadratic equation is ax 2  bx  c  0 , you divide


b c
this later equation by a throughout, you get: x 2  x   0 . On comparing the two equations, you will
a a
get:
b c
   and  
a a

Example: without solving, form a quadratic equation whose roots are the squares of the roots of the

equation: 2 x  x  6  0
2

Solution: Let  and  be the roots of the given equation 2 x  x  6  0 . You compare the equation
2

1
x 2  (   ) x    0 , with the given equation with 1 as the coefficient of x 2 , x 2  x  3  0 and
2
1
then you will have,      , and   3 . Since (   )    2   , the sum of the roots
2 2 2

2
25
for the new equation is,  2   2  , and the product is, ( )  9 . Therefore, the required equation
2

4
25
is x 2  x  9  0 , which is 4 x 2  25 x  36  0 .
4
Stop and think: do you now know the quadratic formula, the relationship between the roots of a quadratic
equation, if so, try the following problems and make sure you get the answers given below.

PROBLEMS
1. Given that the equation kx2 + 12x + k = 0, where k is a positive constant, has equal roots, find the value of
k.
2. The equation 2x2 − 3x − (k + 1) = 0, where k is a constant, has no real roots. Find the set of possible values
of k.
3. The equation x2 + kx + (k + 3) = 0, where k is a constant, has different real roots. Find the set of possible
values of k.
4. The equation x2 + kx + 8 = k has no real solutions for x. find the set of possible values of k.
Solutions
1. k = 6

Page 56 | H A M A C H I L A . I
2.
3. k < −2, k > 6
4. −8 < k < 4

APPLICATIONS OF QUADRATIC
If 1200 cm2 of material is available to make a box with a square base and an open top, find the largest
possible volume of the box.
Solution. Let s be the length of the side of the square base; let h be the height of the box; let V be the volume
of the box. Then V = hs2. We want to maximize V , so we need to write h in terms of s (or s in terms of h).
The surface area of the box is
(area of base) + 4 · (area of one side) = s2 + 4hs,
so 1200 = s2 + 4hs. We can solve this equation for h in terms of s:

Plugging this into the expression for V , we get . Therefore,

,
and this is V as a function of one variable, s. What is its domain? The volume cannot√

be negative, which means 300 0; solving this inequality for s yields s ≤ −20 3
√√
or 0 ≤ s ≤ 20 3. But s ≥ 0 rules out the possibility of s ≤ −20 3, so the condition is
√h √i
0 ≤ s ≤ 20 3. Therefore, the domain is 0,20 3 . (There are other ways of getting the righthand endpoint of
this domain, or other reasonable choices of righthand endpoint.)

Now we need to find the absolute maximum of on this domain. The derivative is

, and this is never undefined, so we can find the critical numbers by setting it to zero:

,
but −20 is not in the domain, so the only critical number is s = 20.
Now we can use the “closed-interval method” to determine the absolute maximum of V . We test the
critical number and the endpoints of the domain:

V (0) = 0; = 4000; .

Page 57 | H A M A C H I L A . I
The highest of these is V (20) = 4000, so the absolute maximum is 4000. Therefore, the largest possible
volume is 4000 cm3. (This is achieved when the base side length is s = 20 cm

and the height is = 10 cm.)

POLYNOMIALS
Introduction
A functions of the form f ( x)  ao x n  a1 x n 1  a2 x n  2  ....  an 1 x  an , where ai , for i  1,2,3,..., n

constants and n is a positive integer. This is called a polynomial function of order n. If you set f ( x)  0 ,

then we have ao x n  a1 x n 1  a2 x n  2  ....  an 1 x  an  0 , this called a polynomial equation of order n.

If you have a function of the form: f ( x)  3x  2 x  x  1 this is a polynomial of order 5. And if you
5 3

have an equation of the form 3x 4  2 x 3  x  1  0 this is a polynomial equation of order 4. Note that a
polynomial of order n has a finite sum of n terms. Our task on polynomials is either to sketch if you have a
polynomial function or solving for the polynomials roots if you have a polynomial equation. To do this we
shall begin by looking at polynomial expression called algebraic expressions. We should look at how to
divide expression by another expression.

Division of algebraic expressions


Just as you divide any real number by another real, such as 345 divided by 34, you carried out the operations

as follows: 34 342 this can also be done when dividing algebraic expressions.

Long division: If f(x) is divided by d(x) and d(x) is non-zero, and the degree of d(x)  the degree of f(x),
then two unique polynomials q(x) the quotient and r(x) the remainder exist, this is called improper rational
expression , so that:
f ( x) r ( x)
 q ( x) 
d ( x) d ( x)
Note - the degree of r(x) is less than the degree of d(x), and this called proper rational expression.

5 x 3  x 2  3x  2
Example: Find the quotient and remainder when
x2  2x  5

Page 58 | H A M A C H I L A . I
6 x  53
Therefore, q ( x)  5 x  11 is the quotient and r ( x)   is the remainder.
x  2x  5
2

The Remainder Theorem: If a polynomial f(x) is divided by (x-a), where a is any constant, until a constant
remainder independent of x is obtained, this remainder is equal to f (a).

Example: Find the remainder when (2x3+3x+x) is divided by (x+4).

The reader may wish to verify this answer by using algebraic division.

The Factor Theorem: (a special case of the Remainder Theorem) . If f (a) = R is zero, then, 𝑎 is a zero of
f(x), then (x−a) is a factor of the polynomial f(x).
The converse is true, that is, If (x-a) is a factor of f(x), then f(a) = R = 0, and a is a zero of f(x).

Page 59 | H A M A C H I L A . I
Example:

Synthetic division: You can determine the quotient and remainder when an expression f(x) is divided by a
factor x-a, where a is a constant using a method called synthetic division.
The process of division for polynomial in x (or any one letter) may be greatly simplified when the divisor
is in the form x – a. This process, known as synthetic division, will be illustrated.

Example: Divide 5 x 3  14 x  3 by x – 2 using synthetic division.


Solution: You write the coefficients of the expression as follows:

The procedure is as follows: after writing the coefficients of the expression, you find a, in this case a = 2.
You write the division upside down, drop 5 the coefficient of the highest power, multiply this by a, in this
case by 2, write the answer under the next coefficient, add write the answer below, in this case 10, multiply
your answer by 2, write the answer, 20 under the next coefficient, add, your answer is 6, multiply 6 by 2
and write your answer under the constant 3, add, you will get 15. The number 15 is the remainder, while

Page 60 | H A M A C H I L A . I
your quotient is: 5 x 210 x  6 and remainder 6. This is easier and faster in find the quotient and remainder,
than the long division.
Fundamental Theorem of algebra: Every polynomial function whose defining equation is
f ( x)  ao x n  a1 x n 1  a2 x n  2  ....  an 1 x  an

For n  1 and a0  0 , has at least one (real or complex) root or at most n roots.

Stop and think: what is a polynomial, what are the zeros of a polynomial, can you perform a long division
given two polynomial, and are you able to perform synthetic division? Have you understood the Remainder
theorem/ Factor theorem? If so follow nicely the way question one has been done for you and then try the
rest.
EXERCISE
1. Factorise the expression x3−17x2+54x−8 given that one of the factors is (x−4).

Solution
Gi Given that x −4 is a factor we can write x3 −17x2 +54x −8 = (x −4)× (a quadratic polynomial)
The polynomial must be quadratic because the expression on the left is cubic and x −4 is linear. Suppose we
write this quadratic as ax2 + bx + c where a, b and c are unknown numbers which we need to find. Then
x3 −17x2 +54x −8 = (x −4)(ax2 + bx + c)
Removing the brackets on the right and collecting like terms together we have
x3 −17x2 +54x −8 = ax3 +(b −4a)x2 +(c −4b)x −4c
Like terms are those which involve the same power of the variable (x).
Equating coefficients means that we compare the coefficients of each term on the left with the corresponding
term on the right. Thus if we look at the x3 terms on each side we see that x3 = ax3 which implies a must equal
1. Similarly by equating coefficients of x2 we find −17 = b−4a With a = 1 we have −17 = b − 4 so b must
equal −13. Finally, equating constant terms we find −8 = −4c so that c = 2.
As a check we look at the coefficient of x to ensure it is the same on both sides. Now that we know a = 1,
b = −13,c = 2 we can write the polynomial expression as x3 −17x2 +54x −8 = (x −4)(x2 −13x +2)

2. Let the polynomial expression x3 −17x2 +54x −18 be denoted by P(x). Verify that x = 4 is a solution of the
equation P(x) = 0. Hence find the other solutions.
3. Plot a graph of the function y = 4x4 −15x2 +5x +6 and hence approximately solve the equation 4x4 −15x2
+5x +6 = 0.

Page 61 | H A M A C H I L A . I
Solution
We substitute x = 4 into the polynomial expression P(x):
P(4) = 43 −17(42)+54(4)−8 = 64−272+216−8 = 0
So, when x = 4 the left-hand side equals zero. Hence x = 4 is indeed a solution. Knowing that x = 4 is a root
we can state that (x−4) must be a factor of P(x). Therefore P(x) can be re-written as a product of a linear and
a quadratic term:
P(x) = x3 −17x2 +54x −8 = (x −4)× (quadratic polynomial)
The quadratic polynomial has already been found in a previous task so we deduce that the given equation can
be written
P(x) = x3 −17x2 +54x −8 = (x −4)(x2 −13x +2) = 0 In this form we see that x −4 = 0 or x2 −13x +2 = 0
The first equation gives x = 4 which we already knew.
The second equation must be solved using one of the methods for solving quadratic equations given in
Section 3.2. For example, using the formula we find
√2
− b± b − 4ac
x = a =1 b = − 13 c =2
2a
13 ± √ ( − 13)2 − 4. 1. 2
=
2
13 ± √161 13 ± 12. 6886
= =
2 2
So x = 12.8443 and x = 0.1557 are roots of x2 −13x +2.
Hence the three solutions of P(x) = 0 are x = 4, x = 12.8443 and x = 0.1557, to 4 d.p.

Page 62 | H A M A C H I L A . I
y

x
−5 5

y = 4 x 4 − 15x 2 +5 x +6

x = 1 x = 1 . 5 x = − 0. 5 x = − 2

An important feature of the graph of a polynomial is that it is continuous. There are never any gaps or
jumps in the curve. Polynomial curves never turn back on themselves in the horizontal direction, (unlike a
circle). By studying the graph in Figure 6 you will see that if we choose any two values of x, say a and b,
such that y(a) and y(b) have opposite signs, then at least one root lies between x = a and x = b.

PROBLEMS
1. Factorise x3 − x2 −65x −63 given that (x +7) is a factor.
2. Show that x = −1 is a root of x3+11x2+31x+21 = 0 and locate the other roots algebraically.
3. Show that x = 2 is a root of x3 −3x −2 = 0 and locate the other roots.
4. Solve the equation x4 −2x2 +1 = 0.
5. Factorise x4 −7x3 +3x2 +31x +20 given that (x +1) is a factor.

Page 63 | H A M A C H I L A . I
6. Given that two of the roots of x4 +3x3 −7x2 −27x −18 = 0 have the same modulus but different sign, solve
the equation.
(Hint - let two of the roots be α and −α and use the technique of equating coefficients).

ANSWERS
Answers
1. (x +7)(x +1)(x −9)
2. x = −1,−3,−7
3. x = 2,−1 (repeated)
4. x = −1,1 (each root repeated)
5. (x +1)2(x −4)(x −5)
6. (x +3)(x −3)(x +1)(x +2)

Equations with Radicals


The title seems to imply that we’re going to look at equations that involve radicals. However, we are going
to restrict ourselves to equations involving square roots. The techniques involved here can be used to solve
equations with other radicals, however the work is usually significantly messier than when dealing with
square roots. So in this section, we will work with square roots only.

Before proceeding it should be mentioned that topic that include equations reducible to quadratic form will
be discussed separately. The reason is that we will in fact end up solving a quadratic equation in most
cases. However, the approach is significantly different and so we’re going to separate the two topics into
different sections in this book.

Example: Solve x  x  6 x  0
Solution
Here the basic problem is how to remove the square root. If there was no square root, we would solve the
equation very easily. The first thing that we need to do is to remove the square root. To do that we transpose
the term with a square root to the right, as follows. If you square the equation both sides as it is, the square
root will not go. This is how it is done;

Page 64 | H A M A C H I L A . I
x x6 0
x  x6
( x) 2  ( x  6 ) 2
x2  x  6
x2  x  6  0
( x  3)( x  2)  0
x  3 or x  2
Not that squaring both side produces a quadratic equation which when factorized two values x = 3 and x =
-2 are found as solution to a quadratic equation and not radical equation. Find the solution by substituting
these values in order to find the value which satisfies the original equation.
That is if x = 3, the equation become,

x  x6
3  36
3 9
33
Which is true.
That is if x = -2, the equation become,

x  x6
2  26
2 4
2 2
Which is false.

In this case the solution is x = 3. Note that x = -2 is not one of the solution as it does not satisfies the
original equation.
Note that when squaring both sides of the equation the original equation is changed and in the process
introduced a solution that is not a solution to the original equation. With these problems it is very important
that the solutions that satisfies the original equation. Take the values that are actual solutions to the original
equation. Care must be taken when solving such equations. When solving the quadratic equation two
solutions are generally found and it is possible both of these, or one of these, or none of these values will
be the solution(s) to the original equation. The only way to know is to check your solutions!

Page 65 | H A M A C H I L A . I
Example: Solve each of the following equations

(a) y  y4  4
Solution
If you square both sides you are likely to encounter problems. Before discussing the problem we’ve got
here let’s make sure you can do the squaring that we did above since it will show up on occasion. All that

we did here was use the formula (a  b)  a  2ab  b , ( y  y  4 ) 2  (4) 2 Implies a  y and
2 2 2

b y4
When we square both sides the there is still a square root in the problem and we’ve made the remainder of
the problem messier as well. So, what we’re going to need to do here is make sure that we’ve got a square
root all by itself on one side of the equation before squaring. Once that is done we can square both sides
and the square root really will disappear. The easiest way is to transpose y to the right as follows and square
both sides.

y y4  4
y4  4 y
( y  4 ) 2  (4  y ) 2
y  4  16  8 y  y 2
y 2  9 y  20  0
( y  5)( y  4)  0
y  5 or y  4
When checking which solution satisfies the origin equation, we see that;
For x = 5, we have

y y4  4
5 54  4
53 4
8 is not equal to 4, Therefore, x = 5 is not a solution.
For y = 4, we have

y y4  4
4 44  4
44
Which is true. Therefore, x = 4, is the solution to the original equation.

Page 66 | H A M A C H I L A . I
(b) 1  x  2x  3

Solution.

1  x  2x  3
Now we can solve this problem fast as follows.
Transpose x to the left and square both sides

1  x  2x  3
1  x  2x  3
(1  x) 2  ( 2 x  3 ) 2
1  2x  x 2  2x  3
x 2  4x  4  0
( x  2)( x  2)  0
x2
If x = 2, then

1  x  2x  3
1 2 43
1  2 1
1 3
Which is false.
So, x = 2 isn’t a solution to the original equation. Since there is only one possible solution, this means that
there are no solutions to the original equation. This doesn’t happen too often, but it does happen at times.

(c) 4z  3  1  z
Solution:
Transpose 1 to the right as follows and square both sides;

5z  6  2  z
5z  6  2  z
( 5 z  6 ) 2  ( z  2) 2
5z  6  z 2  4 z  4
z2  z  2  0
( z  2)( z  1)  0
x  2 or x  1

Page 67 | H A M A C H I L A . I
Checking the solutions to this quadratic equations, we have;
If z = 2, we have

5z  6  2  z
10  6  2  2
42  2
22
Which is true, so x = 2 is a solution to the original equation.
If z = -1, we have

5z  6  2  z
56 2  2
1 2  2
1  2
Which is false, therefore, z = -1 is not a solution to the original equation.

The examples given above has shown that once we get our list of possible solutions anywhere from none
to all of them can be solutions to the original equation. Always remember to check your answers!

So far, all the equations that we’ve looked at have had a single square root in them. Now we look at
problems with more than one square root in the equations. In the following examples with more than one
square root, we need to get a square root all by itself on one side of the equation before squaring. Once that
is done we can square both sides and the square root really will disappear.
So, when there is more than one square root in the problem we need to check whether our possible solutions
satisfies the original equation. It is possible that anywhere from none to all of the possible solutions will in
fact be solutions and the only way to know for sure is to check them in the original equation.

Example 3. Solve each of the following

(a) 2x  1  x  4  2
(b) x  7  2  3 x
Solutions

(a) 2x  1  x  4  2

Page 68 | H A M A C H I L A . I
Before squaring both sides, make sure that we do not have two square roots on one side of the
equation. If you square both sides with two square roots on one side, you will complicate the
problem.
To this problem, we transpose one square root to the rigxht so that we have one square root on each
side, and square both sides. So we have the following;

2x  1  x  4  2
2x  1  x  4  2
( 2 x  1) 2  ( x  4  2) 2
2x  1  x  3  4  4 x  4
x2 4 x4
( x  2) 2  (4 x  4 ) 2
x 2  4 x  4  16( x  4)
x 2  20 x  68  0
(b) x  7  2  3 x
x 7  2  3 x
( x  7  2) 2  ( 3  x ) 2
x 7 4 4 x 7  3 x
x  11  4 x  7  3  x
4 x  7  8  2 x
(4 x  7 ) 2  (8  2 x) 2
16( x  7)  64  4 x 2  32 x
16 x  112  64  4 x 2  32 x
4 x 2  16 x  64  112  0
4 x 2  16 x  48  0

INEQUALITIES
Expressions involving inequality signs such as <, >, or ,  are called inequalities. These type of
inequalities do not demand one value as an answer but a set of values which will satisfy the inequality.
Linear Inequalities

Page 69 | H A M A C H I L A . I
Inequalities of the form ( x  a )  0 or ( x  a)  0 are called linear inequalities. Our problem is to find
the set of values that satisfied the inequality.
Example: Find the solution set for the following inequality, ( x  1)  0 .

Solution: We solve the inequality as follows: ( x  1)  0


x  1
The solution set is x : x  1, x  R, this is written in set builder notation

Quadratic Inequalities.
Inequalities of the form ax 2  bx  c  0 are called quadratic inequalities because they involve quadratics.
Inequalities of this form require you to factorize the expression on the left hand side. After factorizing then
you follows the following procedures in order for you get the set(s) of values that satisfies the inequality.
Example: Find the solution set for the inequality x 2  3x  2  0
Solution: You begin by factorizing the quadratic expression as follows;
x 2  3x  2  0
( x  2)( x  1)  0
To find the solution set of this inequality, you find the critical values of the inequality by solving for x. In
this case x = -2 or -1. You make a table showing the critical points as shown below: Take a number in the
given interval and substitute in the factor and write the sign in the box as indicated, and the last row which
is the product of the two factors will determine the solution depending on the inequality given in the
question. In this case, you want less than or equal so your solution will be the interval with a negative in
the last row.

The table above shows that the sets of point which satisfies the given quadratic inequality is read from the

last row, ie the shaded part, which is x : 2  x  1, x  R.

Page 70 | H A M A C H I L A . I
This does not matter how many factors that are being multiplied, all what is needed is to find the
critical value of each factor and this will give regions where to check whether the factor is positive or
negative and assign them to each region and follow the procedure as given above.

Example: Find the solution set of ( x  1)( x  1)( x  2)  0


Solution: Here we find the values of x for which this inequality vanishes. These values are called critical
values. Here we have x = 1, -1, and 2.
So we make a table as follows:

The solution set to the given inequality is {x : 1  x  1, x  R}  {x : x  2, x  R}

x 1
Example: Find the solution set for the following inequality 2
x 1
Solution:
To find the solution set of this problem, you first transpose 2 to the left because you cannot cross multiply
since the denominator is a variable. Note that when you multiply by a negative quantity the sign changes,
so x is a variable and it can assume a negative number or a positive number, but we do not know for which
values this x stands for as it is a variable. So we proceed as follows:

Page 71 | H A M A C H I L A . I
x 1
2
x 1
x 1
20
x 1
x  1  2( x  1)
0
x 1
x  1  2x  2
0
x 1
 x3
0
x 1
x3
0
x 1
Note the change of the sign from greater than zero to less than zero.

You now find the critical values of the numerator and denominator and proceed as example 1 given above.
The critical values are: x  3 and x  1

We hope you have enjoyed inequalities, they are the simplest part of this mathematics but yet very useful
at all times in life and so they are always examinable.

ABSOLUTE VALUE INEQUALITIES

The absolute value of a quantity represents the distance that the value of the expression is from zero on a
number line. So |𝑥 − 𝑎| = 𝑏, where 𝑏 > 0, say that the quantity 𝑥 − 𝑎 is 𝑏 units from 0, 𝑥 − 𝑎 is 𝑏 units to
the right of 0 or 𝑥 − 𝑎 is 𝑏 units to the left of 0. When we say |𝑥 − 𝑎| > 𝑏, 𝑏 > 0, then 𝑥 − 𝑎 is at a distance
from 0 that is greater than 𝑏. Thus, 𝑥 − 𝑎 > 𝑏 𝑜𝑟 𝑥 − 𝑎 < −𝑏. Similarly, if |𝑥 − 𝑎| < 𝑏, 𝑏 > 0,

Page 72 | H A M A C H I L A . I
then 𝑥 − 𝑎 is at a distance from 0 that is less than 𝑏. Hence, 𝑥 − 𝑎 is between b units below
0, −𝑏 𝑎𝑛𝑑 𝑏 𝑢𝑛𝑖𝑡𝑠 𝑎𝑏𝑜𝑣𝑒 0.

Examples: solve each of these inequalities for x.

(a) |𝑥 − 3| > 4
Solution:
|𝑥 − 3| > 4, 𝑡ℎ𝑒𝑛 𝑥 − 3 > 𝑜𝑟 𝑥 − 3 < −4
𝑡ℎ𝑢𝑠, 𝑥 > 7 𝑜𝑟 𝑥 < −1.
𝑡ℎ𝑒 𝑠𝑜𝑙𝑢𝑡𝑖𝑜𝑛 𝑖𝑛𝑡𝑒𝑟𝑣𝑎𝑙 𝑖𝑠 (−∞, −1) ∪ (7, ∞)
(b) |𝑥 + 4| < 7
Solution:
|𝑥 + 4| < 7 then – 7 < 𝑥 + 4 < 7
Thus, −11 < 𝑥 < 3
𝑡ℎ𝑒 𝑠𝑜𝑙𝑢𝑡𝑖𝑜𝑛 𝑖𝑛𝑡𝑒𝑟𝑣𝑎𝑙 𝑖𝑠 (−11, 3)
(c) |𝑥 − 5| < −3
Solution:
Since the absolute value of a number is always greater than or equal to zero, there are no values for
which the absolute value will be less than −3. Thus, there is no solution and we may write ∅ for
the solution interval.
(d) |𝑥 − 5| > −5
Solution:
Since the absolute value of a number is always at least zero, it is always greater than −5. Thus, the
solution is all real number, and the solution interval we write (−∞, ∞).
(e) |2𝑥 + 1| < 5
Solution:
|2𝑥 + 1| < 5
−5 < 2𝑥 + 1 < 5
−6 < 2𝑥 < 4
−3 < 𝑥 < 2
The solution set is the interval (−3, 2)
(f) |−3𝑥 − 2| ≤ 6
Solution:
|−3𝑥 − 2| ≤ 6

Page 73 | H A M A C H I L A . I
−6 ≤ −3𝑥 − 2 ≤ 6
−4 ≤ −3𝑥 ≤ 8
4 8
≥𝑥≥−
3 3
4 8 8 4 8 4
The statement ≥ 𝑥 ≥ − is equivalent to − ≤ 𝑥 ≤ . Therefore the solution set is [− , ].
3 3 3 3 3 3
𝑥−2
(g) | | < 4
𝑥+3

Solution:
𝑥−2 𝑥−2
|𝑥+3| becomes −4 < 𝑥+3 < 4, which can solved as
𝑥−2 𝑥−2
> −4 𝑎𝑛𝑑 <4
𝑥+3 𝑥+3
𝑥−2 𝑥−2
+4>0 𝑎𝑛𝑑 −4<0
𝑥+3 𝑥+3
𝑥 − 2 + 4(𝑥 + 3) 𝑥 − 2 − 4(𝑥 + 3)
>0 𝑎𝑛𝑑 <0
𝑥+3 𝑥+3
𝑥 − 2 + 4𝑥 + 12 𝑥 − 2 − 4𝑥 − 12
>0 𝑎𝑛𝑑 <0
𝑥+3 𝑥+3
5𝑥 + 10 −3𝑥 − 14
>0 𝑎𝑛𝑑 <0
𝑥+3 𝑥+3
Finding the critical points, you equate the numerator to zero and the denominator to zero then solve
for 𝑥. You will have the solution sets for the inequalities above as:

) (
) (
−14 −3
−3 −2
3

The intersection of the two solution sets is

) (
−14 −2
3
𝑥−2 −14
∴ the solution set of |𝑥+3| < 4 is (−∞, 3
)∪ (−2, ∞)

Exercise:
Solve each inequality and express the solution set in interval form.
𝑥+1
1. >0
𝑥−5
2𝑥−1
2. 𝑥+2
<0

Page 74 | H A M A C H I L A . I
3. |𝑥| < 6
𝑛
4. ≥3
𝑛+2
2 3
5. >
𝑥+1 𝑥−4

6. |𝑥| ≥ 4
𝑥−1
7. |𝑥−4| < 2
𝑥+4
8. |𝑥−5| ≥ 3

Page 75 | H A M A C H I L A . I
DIFFERENTIATIONS

Introduction

This unit begins with the background of the development of calculus by Isaac Newton (1642-1727) and
Gottfried Wilhelm Leibniz (1646-1716). Their investigation on the slope of a tangent on curves at a given
point and finding the areas of irregular shapes in a plane gave birth to differential and integral calculus
respectively.

The unit also gives an overview of the linkage between pre-calculus mathematics with differential calculus
just to show you that differential calculus is not independent of pre-calculus mathematics.

Historically, the development of calculus by Isaac Newton (1642-1727) and Gottfried Wilhelm Leibniz
(1646-1716) resulted from the investigation of the following problems:

(i) Finding the tangent line to a curve at a given point on the curve
(ii) Finding the area of a planar region bounded by an arbitrary cure

The tangent line problem might appear to be unrelated to any practical applications of mathematics, but as
you will see later, the problem of finding the rate of change of one quantity with respect to another is

Page 76 | H A M A C H I L A . I
mathematically equivalent to the geometric problem of finding the slope of the tangent line to a curve at a
given point on the curve. It is precisely the discovery of the relationship between these two problems that
spurred the development of calculus in the seventeenth century and made it such an indispensable tool for
solving practical problems. The following are the few examples of such problems:

1. Finding the rate of change of a bacteria population with respect to time


2. Finding the velocity of an object
3. Finding the rate of change of a company’s profit with respect to time
4. Finding the rate of change of a travel agency’s revenue with respect to the agency’s expenditure
for advertising.
The study of tangents-line problem led to the creation of differential calculus, which relies on the concept
of the derivative of a function. The study of the area problem led to the creation of integral calculus, which
relies on the concept of the ant-derivative or integral, of function. Both the derivative of a function and the
integral of a function are defined in terms of a more fundamental concept called – Limit.

What is Calculus?
Calculus can be referred to as a “ Limit Machine” that involves three stages. The first stage is pre-calculus
mathematics such as the slope of a straight line or area of a rectangle. The second stage is the limit process
and the third stage is new calculus formulation, such as derivative of integral.

Students who learn calculus as a collection of new formulas, and reduce calculus to the memorization of
differentiation and integral formulas miss a great deal of understanding and lacks self confidence and
satisfaction of the subject matter. As a result they may not appreciate the applications of calculus in
modeling real life situations in economics, business and engineering. In this text, our goal is to learn how
pre calculus formulas and techniques are used as building blocks to produce the more general calculus
formulas and techniques.

Page 77 | H A M A C H I L A . I
Pre calculus Mathematics With Differential calculus

Average rate of change between t=a and t=b Instantaneous rate of change at t=a

Curvature of a circle Curvature of a curve

5.

Tangent plane to a sphere

Tangent plane to a surface

Direction of a motion Direction of motion along

Page 78 | H A M A C H I L A . I
Along a straight line A curved line.

Figure 6.3.3(a) Figure 6.3.3(b)

These are some of the examples of pre-calculus formulas required to understand differential calculus
formulas. In the study of calculus, what happens to the value of a function as the independent variable gets
very close to a particular value is very important. We came across this concept in the introduction where
you zoomed in on a curve to get an approximation for the slope of that curve. This lead to the theory of
limits and continuity as x approaches a particular number or approaches a number from either the left or
right.

6.4 Limits

In this unit you will learn how to take limits. As consecutive points, given by the terms of the

1
Sequence 2  as shown on a number scale, it is noted that they cluster about the point 2.
n

In such a way that there are points of the sequences whose distance from 2 is less than any pre assigned
positive number, however small. In this unit we shall be interested in the limit of a function. Let x  2 ,

then f ( x)  x  4 as limit. Under this assumption, we say “the limit, as x approaches 2, of x is 4” and
2 2

write lim x  4
2
x 2

Right and Left limits: as x  2 over the sequence given above, its value is always less than 2. We say

 1
that x approaches 2 from the left and write x  2 . Similarly, as x  2 over the given sequence 2  ,
n

its value is always greater than 2. We say that x approaches 2 from the right and we write x  2 . Clearly,
the statement lim f ( x ) exists implies that both the left lim f ( x) and the right limit lim f ( x) exists and
x a xa xa

are equal. However, the existence of the right (left) limits does not imply the existence of the left (right)
limit.

Page 79 | H A M A C H I L A . I
Example

The function f ( x)  9  x has the interval  3  x  3 as domain of definition. If a is any number on


2

the open interval  3  x  3 , then lim 9  x exists and is equal to 9  a . Now consider a = 3. First,
2 2
xa

let x approaches 3 from the left, then lim 9  x 2  0 . Next, let x approaches 3 from the right, then
x 3

lim 9  x 2 does not exist since for x>3, 9  x 2 is imaginary. Thus, lim 9  x 2 does not exist.
x 3  x 3

Theorems: The following theorems on limits are listed for future reference.

1. If f(x)=c, a constant , then lim f ( x)  c


xa

If lim f ( x)  A and lim g ( x)  B , then


xa xa

2. lim k . f ( x)  kA , k being a constant


xa

3. lim [ f ( x)  g ( x)]  lim f ( x)  lim g ( x)  A  B


x a x a x a

4. lim [ f ( x).g ( x)]  lim f ( x). lim g ( x)  A.B


x a xa xa

f ( x) lim f ( x) A
5. lim  x a  ,B  0
x a g ( x) lim g ( x) B
x a

6. lim n f ( x)  n lim f ( x)  n A
x a x a

These results may be very useful in the next sections.

Limits as x Approaches a Particular Number

Sometimes, finding the limiting value of an expression means simply substituting a number directly in the
function..

Example

Find the limit as t approaches 10 of the expression P = 3t + 7.

Page 80 | H A M A C H I L A . I
Answer, we write this using limit notation as: lim (3t  7)
t 10

In this example there is no complication - we simply substitute and write lim (3t  7  37
t 10

There is no complication because f(t) = 3t + 7 is a continuous function. But there are cases where we cannot
simply substitute like this.

Example 6.4.3

We know that x cannot equal 3 in the following expression (because we cannot have a denominator equal

x 2  2x  3
to zero): f ( x) 
x3

However, we can see that the function approaches a particular value as x approaches 3 from the left:

x 2.5 2.6 2.7 2.8 2.9

f(x) 3.5 3.6 3.7 3.8 3.9

Continuing, we get closer and closer to x = 3:

x 2.9 2.92 2.94 2.96 2.97 2.98 2.99

f(x) 3.9 3.92 3.94 3.96 3.97 3.98 3.99

x 3.5 3.1 3.01 3.00001

f(x) 4.5 4.1 4.01 4.00001


Table 6.9.1

Likewise, approaching x = 3 from the right gives the same limit value: We note that the function value is

x 2  2x  3
getting closer and closer to 4. We write: lim 4
x 3 x3

NOTE: We could have evaluated this limit by factorizing first:

CAUTION: The factorizing process is only possible in this example because we have:

Page 81 | H A M A C H I L A . I
x ≠ 3. This is a typical problem in the study of introductory limits. It appears to be a bit trivial, in that we
could have factored it, cancelled and substituted x = 3 like we just saw. But the example is important for
the concept that there is no actual value of the function when x = 3, but if we get really, really close to 3,
the function value is really close to some value (4, in this case).

Note that if you get the form 0 when you substitute directly in the function, this form is called
0
indeterminate, means you cannot determine the limit, so you use factorization method to cancel one of the
zeros, generally the zero in the denominator but not always.

Example 6.4.3

x 2  25
Find the limit lim 2
x 5 x  x  30

Solution Although the limit in question is the ratio of two polynomials, x = 5 makes both the numerator
and denominator equal to zero. We need to factor both numerator and denominator as shown below.

Simplify to obtain

10

11

Example 6.4.4

x2 1
Find the limit lim
x 1 x 1
Solution If you substitute x = -1 directly in the function, you will get 0 and this form is called
0
indeterminate, meaning you cannot determine the limit of the function as x approaches -1.
So we factorize the numerator and simplify the expression and then take limit.
x 1
2
( x  1)( x  1)
lim  lim
x 1 x 1 x 1 ( x  1)

Page 82 | H A M A C H I L A . I
lim ( x  1)  2
x 1

Limits as x Approaches Infinity

When taking limits as x approaches infinity, written as; lim f ( x ) , and we find the form  this form is
x  
also indeterminate. In this case we cannot factorize into factors as shown in the two examples above, what

1
we need to do is factor out the highest power of x, and use the fact that lim    0 .
x 
 x
Example 6.4.5
5  3x
Find the limit of lim ( ).
x  6x  1
Solution: This time it is not so obvious what the limit value is. We could substitute larger and larger values
of x until we could see what was happening (try 100, then 1000, then 1000000 and so on). Or, we could

1
rearrange the expression and use the fact that lim    0 ,to find the limiting value. We divide throughout
x 
 x
by x to get the expression in a form where we can evaluate it.

 5 3
 5  3x 
lim    lim x 

x   6 x  1 x 
  6 1
 x
03

60
1

2
 5  3x 
Notice that we cannot substitute ∞ into the fraction   , because this does not make mathematical
 6x  1 
 5  3() 
sense. Please do not write   . It has no meaning
 6(  )  1 
Example 6.4.6

3x 2
Find the limit lim 2
x  4 x  2 x  1

Solution. Factor x 2 in the denominator and simplify.

Page 83 | H A M A C H I L A . I
2
As x takes large values (infinity), the terms 2/x and 1/x approaches 0 hence the limit is
3

4

Example 6.4.7
x 1
Find the lim
x  2x 2  3
Solution. Factor x 2 in the numerator and denominator and simplify.

As x takes large values (infinity), the terms 1/x and 1/x 2 and 3/x 2 approaches 0 hence the limit is

0
 0
2

DERIVATIVES

Introduction. In this unit we begin by introducing the concept of differentiation using the concept of slope
of a straight line. This will be developed to derivative of a function from first principles which illustrate the
ideas of limits which we discussed in the previous unit. Then we will discuss differentiation of functions
using basic rules, and finally we shall discuss some examples of the applications of differentiations in
economics and businesses. Differentiations can also be applied in areas such as: Temperature change at a
particular time, Velocity of a falling object at a particular time, Current through a circuit at a particular
time, Variation in stock market prices at a particular time, Population growth at a particular time,
Temperature increase as density increases in a gas.

The Slope (gradient) of a Tangent to a Curve (Numerical Approach)


Since we can model many physical problems using curves, it is important to obtain an understanding of the
slopes (gradient) of curves at various points and what a slope means in real applications. Remember: We
are trying to find the rate of change of one variable compared to another. In this section, we show you one

Page 84 | H A M A C H I L A . I
of the historical approaches for finding slopes (gradient) of tangents, before differentiation was developed.
This is to give you an idea of how it works. Before we proceed, let look at a numerical example given below
just as a motivation into this unit.
Example6.5.1

Find the slope (gradient) of the curve f(x) = x2 at the point (2,4), using a numerical method.

Solution. We start with a point Q(1, 1) which is near P(2,4):

y 2  y1 4  1
The slope (gradient) of PQ is given by: m   3
x 2  x1 2  1

Now we move Q further around the curve so it is closer to P. Let's use Q(1.5,2.25) which is closer to P(2,4):

y 2  y1 4  2.25
The slope (gradient) of PQ is now given by: m    3.5
x 2  x1 2  1.5

We see that this is already a pretty good approximation to the tangent at P, but not good enough.

Now we move Q even closer to P, say Q(1.9,3.61). Now we have:

Page 85 | H A M A C H I L A . I
y 2  y1 4  3.61
So m   3.9
x 2  x1 2  1.9

We can see that we are very close to the required slope (gradient). Now if Q is moved to (1.99,3.9601),
then slope PQ is 3.99. If Q is (1.999,3.996001), then the slope is 3.999. Clearly, as x → 2, the slope of PQ
→ 4. But notice that we cannot actually let x = 2, since the fraction for m would have 0 on the bottom, and
so it would be undefined.

We have found that the rate of change of y with respect to x is 4 units at the point x = 2 . We will now
extend this numerical approach so that we can find the slope of any continuous curve if we know the
function. We will see an algebraic approach that can be used for most functions.

The Derivative from First Principles

In this section, we will differentiate a function from "first principles". This means we will start from scratch
and use algebra to find a general expression for the slope of a curve, at any value x.

A first principle is also known as "delta method", since many texts use Δx (for "change in x) and Δy (for
"change in y"). This makes the algebra appear more difficult, so here we use h for Δx instead. We still call
it "delta method". We look at the general case and write our functions involving the familiar x (independent)
and y (dependent) variables.

Page 86 | H A M A C H I L A . I
We wish to find an algebraic method to find the slope of y = f(x) at P, to save doing the numerical
substitutions that we saw in the last section (Slope of a Tangent to a Curve - Numerical approach). We can
approximate this value by taking a point somewhere near to P(x, f(x)), say Q(x + h, f(x + h)).

g
The value is an approximation to the slope of the tangent which we require. We can also write this slope
h
y
as "change in y / change in x" or: m  . If we move Q closer and closer to P, the line PQ will get closer
x
and closer to the tangent at P and so the slope of PQ gets closer to the slope that we want.

If we let Q go all the way to touch P (i.e. h = 0), then we would have the exact slope of the tangent.

Consider figure 6.5.4 given below:

Page 87 | H A M A C H I L A . I
In this work, we write

6. change in y as Δy
7. change in x as Δx

g g f x  h   f ( x)
Now, can be written:  . So also, the slope PQ will be given by:
h h h
y 2  y1 y f ( x  h)  f ( x)
m   . But we require the slope at P, so we let h → 0 (that is let h
x 2  x1 x h
g
approach 0), then in effect, Q will approach P and will approach the required slope. Putting this together,
h
dy f ( x  h)  f ( x )
we can write the slope of the tangent at P as:  lim
dx x0 h

This is called differentiation from first principles, (or the delta method). It gives the instantaneous rate
of change of y with respect to x. This is equivalent to the following (where before we were using h for Δx):
dy y
 lim
dx x0 x

dy f ( x  h)  f ( x )
You will also come across the following for delta method:  lim
dx x0 x

The slope (gradient) of a curve at the point P means the slope (gradient) of the tangent at the point P. Note
that the slope (gradient) of a straight line is the same along the line while the slope (gradient) of a curve is
different at every point along the curve. We need to find this slop (gradient) to solve many applications
since it tells us the rate of change at a particular instant.

y f ( x  h)  f ( x )
And we write  lim and this is called the derivative of f(x) from first principles and
x x  0 h
dy f ( x  h)  f ( x )
this becomes  lim
dx x0 h

Notation for the derivatives

IMPORTANT: The derivative (also called differentiation) can be written in several ways. This can cause
some confusion when we first learn about differentiation. The following are equivalent ways of writing the
dy
first derivative of y = f(x): or f ’(x) or y’.
dx

Page 88 | H A M A C H I L A . I
Example 6.5.2
dy
Find from first principles if y = 2x2+ 3x.
dx
Solution: f(x) = 2x2+ 3x so

We now need to find:

We have found an expression that can give us the slope of the tangent anywhere on the curve.

If x = -2, the slope is 4(-2) + 3 = -5 (red, in the graph below)

If x = 1, the slope is 4(1) + 3 = 7 (green)

If x = 4, the slope is 4(4) + 3 = 19 (black)

We can see that our answers are correct when we graph the curve (which is a parabola) and observe the
slopes of the tangents.

Page 89 | H A M A C H I L A . I
This is what makes calculus so powerful. We can find the slope anywhere on the curve (i.e. the rate of
change of the function anywhere).

Example 6.5.3

a. Find y' from first principles if y = x2 + 4x.

b. Find the slope of the tangent where x = 1 and also where x = -6.

c. Sketch the curve and both tangents.

Solution:

a. Note: y' means "the first derivative". This can also be written dy/dx.

Now f(x) = x2 + 4x

So

b. When x = 1, m = 2(1) + 4 = 6

When x = -6, m = 2(-6) + 4 = -8

c. Sketch:

Page 90 | H A M A C H I L A . I
Example 6.5.4

Find y' from first principles if: f ( x)  x 2 ,

as expected.

dy 1
Example 6.5.5 From first principles find of f ( x) 
dx x

Page 91 | H A M A C H I L A . I
Rules of Derivatives

The good news is that we can find the derivatives of polynomial expressions without using the delta method
that we met in the derivative from first principles. They follow from the "first principles" approach to
differentiating, and make life much easier for us.

d (c ) This is basic. In English, it means that if a quantity has a constant value,


Constant: 0
dx then the rate of change is zero.

d n
n-th power of x: x  nx n 1 This follows from the delta method.
dx

Here, y is some function of x. It means that if we are finding the


Constant d d
(cy)  c ( y ) derivative of a constant times that function, it is the same as finding the
product: dx dx
derivative of the function first, then multiplying by the constant.

Derivative of
d du dv Here, u and v are functions of x. The derivative of the sum is equal to
sum or (u  v)  
dx dx dx the derivative of the first plus derivative of the second.
difference:

Here, u and v are functions of x. The derivative of the product is equal


Derivative of d (uv ) dv du
u v to the first x derivative of the second plus the second x derivative of the
product dx dx dx
first.

du dv Here, u and v are functions of x. The derivative of the quotient is equal


Derivative of v u
d u dx dx to the denominator x derivative of the numerator minus the numerator
quotient ( ) 2
dx v v x derivative of the denominator divided by the denominator squared.

dy
y  ex  ex Special function
dx

Page 92 | H A M A C H I L A . I
dy 1
y  ln x  , x0 Special function
dx x

Function of a Function

If y is a function of u, and u is a function of x, then we say

"y is a function of the function u".

Example 6.5.6 Consider the function

y = (5x + 7)12. If we let u = 5x + 7 (the inner-most expression), then we could write our original function as
y = u12

We have written y as a function of u, and in turn, u is a function of x.

This is a vital concept in differentiation, since many of the functions we meet from now on will be functions
of functions, and we need to recognize them in order to differentiate them properly.

1. Chain Rule

dy dy du
To find the derivative of a function of a function, we need to use the Chain Rule:  .
dx du dx

This means we need to:

1. Recognize u (always choose the inner-most expression, usually the part inside brackets,
or under the square root sign).

2. Then we need to re-express y in terms of u.

3. Then we differentiate y (with respect to u), then we re-express everything in terms of x.

4. The next step is to find du/dx.

5. Then we multiply dy/du and du/dx.

dy
Example 6.5.7 Find if y = (x2+ 3)5.
dx

Page 93 | H A M A C H I L A . I
Solution: In this case, we let u = x2 + 3 and then y = u5. We see that: y is a function of u. u is a function of
dy
x and this means we use the chain rule; for the chain rule, we firstly need to find  5u 4  5( x 2  3) 4
dx
dy
and  2 x . So
dx

dy dy du
 .
dx du dx

 5( x 2  3) 4 (2 x)

 10x( x 2  3) 4

dy
if y  4 x  x .
2
Example 6.5.8 Find
dx
1
Solution: We write the given function as y  (4 x 2  x) , we let u  4 x  x , and our function now
2 2

1 dy dy du dy 1  12 du
becomes y  u 2
. Applying the chain rule  . ,  u and  8 x  1 .Therefore,
dx du dx du 2 dx
dy 1  12
 u .(8 x  1)
dx 2
1 1
 (4 x 2  x) 2 (8 x  1) , since u  4 x 2  x .
2
dy
if y  ( x  5x)
2 6
Example 6.5.9 Find
dx

Page 94 | H A M A C H I L A . I
The Derivative of a Power of a Function (Power Rule)

An extension of the chain rule is the Power Rule for differentiating. We are finding the derivative of
dy dx
x n , n  R (a power of a function): Let y  x n , therefore,  nx n 1 , by the chain rule, give
dx dx
dy
 nx n 1
dx

Example 6.5.10 Find the derivative of y  (2 x 3  1)

dy
 6x 2  0  6x 2
In the case of y  (2 x  1) we have a power of a function. So dx
3

The Product Rule Equation


If u and v are two differentiable functions of x, then the derivative of uv is given by:
dy dv du
y  uv ,  u  v , this can also be written, using 'prime notation' as : y '  (uv) '  u.v '  vu '
dx dx dx
dy
of the given function: y  ( x  1) ( x  1) ;
2 3 3 2
Example 6.5.11 Find
dx

Page 95 | H A M A C H I L A . I
Let u  ( x  1) and v  ( x  1) ; so we have,
2 3 3 2
Solution:

dy
of the given function: y  ( x  4)( x  3) .
2 2
example 6.5.12 Find
dx
Let u  x  4 and v  ( x  3) , so we have;
2 2
Solution.

Page 96 | H A M A C H I L A . I
dy
of the given function: y  ( x  3) (2  x) .
2
Example 6.5.13 Find
dx
1
Let u  ( x  3) and v  ( 2  x ) 2 , so we have,
2
Solution.

The quotient Rule Equation

As with the Product Rule, , if u and v are two differentiable functions of x, then the derivative of u/v is
du dv
v u
u dy
given by: y  dx 2 d x v.This can also be written, using 'prime notation' as :
v dx v
vu '  uv '
y'  .This rule must be remembered, and note that you cannot interchange the positions of these
v2
letters from where they are placed, otherwise the derivative will be wrong. The choice of u and v can be
any of the two given functions. The examples below illustrate this rule.

Page 97 | H A M A C H I L A . I
dy ( x  3) 2
Example 6.5.14 Find of the given function: y 
dx ( x  2) 2

Let u  ( x  3) and v  ( x  2) , so we have;


2 2
Solution;

dy x
Example 6.5.15 Find of the given function y  .
dx (1  x 2 )
1
Solution: Le u  x t and v  (1  x 2 ) 2

Page 98 | H A M A C H I L A . I
1 x2
Find y of the following; y 
'
Example 6.5.16
1 x2
Let u  1  x and v  1  x
2 2
Solution: , so

Applications of Derivatives; Curve Sketching, Determination of Maximum or Minimum of a


function.

In this unit we further explore the power of the derivatives, which we use to help analyse the properties of
functions. The information obtained can then be used to accurately sketch graphs of functions. As we have
seen on numerous occasions, the graph of a function is a useful aid for visualizing the function’s properties
from a practical point of view. The graph of a function also gives at one glance a complete summary of all
the information captured by the function.

Curve Sketching
In curve sketching you need to be Out Comes
Upon completion of this unit you will be able to:
Sketch the curve, showing important features. Avoid drawing x-y boxes and just joining the dots. In this
section we will be using calculus to help find important points on the curve. The kinds of things we will
be searching for in this section are indicated in table 4.1.1:

x - intercept Use y = 0. Note: In many cases, finding x-


intercepts is not easy.

Page 99 | H A M A C H I L A . I
y- intercept Use x = 0

Relative ( local) Maximum dy


Use  0 , sign: x  
dx

Relative ( local ) minimum dy


Use  0 , sign:   
dx

Points of inflection d2y d2y


 0 , and sign of changes
dx 2 dx 2

Table 6.6.1

The maximum or minimum of any curve is the turning point of the curve to give the highest point for the
maximum or the lowest point for the minimum. The turning point can be referred to as ; critical value,
stationary point, extrema. At this point the derivative of the function is equal to zero. Therefore, when
dy
looking for the turning point we solve the equation  0 . The solution(s) of this equation gives the
dx
value(s) of x for which the curve has its maximum or minimum. However, if we solve the equation

d2y
 0 , the solution(s) will give you the values of x where the curve changes from concave down to
dx 2
concave up, and this point is called point of inflection(s).

6.6.2 Increasing and Decreasing Functions and the First Derivative Test

We begin by discussing how derivatives can be used to classify relative extrema as either relative minima
or relative maxima. We begin by defining increasing and decreasing functions.

Definition: Increasing and Decreasing Functions


A function f is increasing on an interval if for any two numbers x1 and x 2 in the interval, x1  x 2 implies

f ( x1  f ( x2 ) . A function f is decreasing on an interval if for any two numbers x1 and x 2 in the interval,
x1  x 2 implies f ( x1  f ( x2 ) . A function is increasing if, as x moves to the right, its graph moves up, and
is decreasing if its graph moves down.

Page 100 | H A M A C H I L A . I
For example, Fig.6.6.2 above the function is decreasing on the interval (, a) , is constant on the interval
(a,b), and is increasing on the interval (b, ). The theorem below states that , a positive derivative implies
that the function is is increasing; a negative derivative implies that the function is decreasing; and zero
derivative on an entire interval implies that the function is constant on that interval.

Theorem 6.6.1 Test for Increasing and Decreasing Functions

Let f be a function that is continuous on the closed interval [a,b] and differentiable on the open interval
(a,b).

8. If f ‘ (x) > 0 for all x in (a,b), then f is increasing on [a,b]

9. If f’ (x) < 0 for all x in (a,b) , then f is decreasing on [a,b]

10. If f ‘ (x) = 0 for all x in (a,b), then f is constant on [a,b]

3 2
Example: 6.6.1 Find the open intervals on which f ( x)  x 
3
x , is increasing or decreasing.
2

Solution Note that f is continuous on the entire real line. To determine the critical numbers of f, we
find f ‘(x) and set it to zero.

f ' ( x)  3 x 2  3 x  0

3x( x  1)  0

X = 0, 1 Critical numbers

Because there are no points for which f ‘is undefined, you can conclude that x = 0 and x = 1 are the only
critical numbers. The following table summarizes the testing of the three intervals determined by these two
critical numbers.

Page 101 | H A M A C H I L A . I
We can show this graphically as follows:

You can see that f is increasing on the interval (,0) and (1, ) , decreasing on the interval (0, 1 ) as
shown on fig 6.6.2.

Guidelines for finding intervals on which a Function is Increasing or Decreasing.

Let f be continuous on the interval (a,b). To find the open intervals on which f is increasing or decreasing,
use the following steps.

11. Locate the critical numbers of f in (a,b), and use these numbers to determine test intervals.

12. Determine the sign of f ‘ (x) at one test value in each of the intervals.

13. Use theorem 4.1.1 to decide whether f is increasing or decreasing on each interval.

These guidelines are also valid if the interval (a,b) is replaced by an interval of the form (, b), ( a, ),
or (, ).

Page 102 | H A M A C H I L A . I
First Derivative Test

After you have determined the intervals on which a function is increasing or decreasing, it is not difficult
3 2
to locate the relative extrema of the function. In Fig. 4.1.2 above, the function f ( x)  x 3  x has a
2
relative maximum at the point (0,0) because f is increasing immediately to the left of x = 0 and decreasing
1
immediately at the point (1, ) because f is decreasing immediately to the left at x = 1 . The following
2
Theorem called the first Derivative Test makes it clear;

Theorem: 6.6.2 The First Derivative test

Let c be a critical number of a function f that is continuous on an open interval I containing c. If f is


differentiable on the interval, except possibly at c, then f9c) can be classified as follows:

14. If f ‘ (x) changes from negative to positive at c, then f(c) is a relative minimum of f.

15. If f ‘ (x) changes from positive to negative at c, then f(c) is a relative maximum of f.

In determining the relative maximum or minimum of a function, we can use another approach, called the
second Derivative Test. This test is based on the fact that if the graph of a function f is concave upward on
an open interval containing c, and f ‘( c) = 0, f(c) must be a relative minimum of f. Similarly, if the graph
of a function f is concave downward on an open interval containing c, and f ‘(c) = 0, f ( c) must be a
relative maximum of f. The second derivative test can be stated without a proof as:

(c.f. Fig. 6.6.3(a),(b) below)

Page 103 | H A M A C H I L A . I
If these two graphs are put together it will look like the figure 6.6.4 below:

dy dy
This method involves finding of the curve. The equation =0 will give you the turning point(s) of
dx dx
the curve, when the value(s) of x is/ are substituted in the original equation y = f(x) , you will get the turning
point, since it will have x- value and y-value , this is a point in the plane. To check whether a particular point

d2y
at x = a is a local (relative) minimum or maximum, we use the second derivative test as follows: Find
dx 2
. At the point x  a ;

Theorem: 6.6.3 Second Derivative Test

Let f be a function such that f ' (a )  0 and the second derivative exists on an open interval containing a.

d2y
(i) If  0 at x = a, then the point at x = a is a local minimum.
dx 2

Page 104 | H A M A C H I L A . I
d2y
(ii) If  0 at x = a, then the point at x = a is a local maximum
dx 2

d2y
(iii) If  0 at x = a, the Test Fails.
dx 2

When the test fails, it means at x = a the point is neither a local minimum nor local maximum, this point is
called an inflexion point., a point where the function changes from concave downward to concave upward.
In this case, you can use the first derivative test.

Example: 6.6.2 Find the relative extreme for

f ( x)  3x 5  5x 3

Solution We begin by finding the critical values of the function f by solving the equation f ‘(a) = 0.

f ' ( x)  15 x 4  15 x 2  15 x 2 (1  x 2 )  0
x   1, 0 ,1

We now find the second derivative

f ' ' ( x)  60x 3  30x  30(2 x 3  x)

You can now apply the Second Derivative Test as follows:

Because the second derivative test fails at (0,0) , you can use the first derivative test and observe that f
increases to the left and right of x = 0. That is, (0,0) is neither a relative minimum nor a relative maximum.

The second derivative can tell us the shape of a curve at any point.

d2y
16. If  0 , the curve will have a minimum-type shape (called concave up)
dx 2

Page 105 | H A M A C H I L A . I
dy d2y
2
Example: 6.6.3 y = x + 3x - 2 has  2 x  3 and  2  0 , for all values of x. So it has a concave
dx dx 2
d2y
up shape for all x. If  0 , the curve will have a maximum-type shape (called concave down)
dx 2

Example: 6.6.4

dy d2y
y = x − 2x + 5 has
3
 3 x  2 and
2
 6 x  0 for all values of x < 0. So it has a concave
dx dx 2
down shape for all x < 0.

Determining Maximum/Minimum using gradient method

A local maximum occurs when y' = 0 and y' changes sign from positive to negative (as we go left to right).

A local minimum occurs when y' = 0 and y' changes sign from negative to positive.

Page 106 | H A M A C H I L A . I
Finding Points of Inflection

A point of inflection is a point where the shape of the curve changes from a maximum-type shape (

d2y d2y
 0 ) to a minimum-type shape (  0 ). Clearly, the point of inflection will occur when
dx 2 dx 2

d2y d2y
 0 and when there is a change in sign (from plus → minus or minus → plus) of .
dx 2 dx 2

Taking the example 1 above, we want to check which point is maximum and which one is minimum, we
proceed as follows: Take the points (-I,8) ans (3,-24) which are the turning points:

d2y d2y
Using the second derivative test , we find . Here  6x  6
dx 2 dx 2

At (-1,8), we substitute x = -1 in the second derivative and get 6(-1)-6= -12 <0, therefore, the point (-
1,8) is a maximum. Check the conditions given above.

At (3,-24) , we substitute x = 3 in the second derivative and we get 6(3)-6 = 12 >0, therefore the point (3,-
24) is a minimum.

Example : 6.7.1. Find the stationary points of the curve whose function is f ( x)  x  3x  9 x  3
3 2

dy dy
Solution We first find of the given function and then solve the equation  0 . The solutions to this
dx dx
equation will give us the values of x where the function has a maximum and a minimum. When these values
of x are substituted in the original function the points obtained are called turning points respectively.
dy dy
 3x 2  6 x  9 , to solve the equation  0 , we set; 3x 2  6 x  9  0 , the solution to this quadratic
dx dx
equation is x   1 or 3 Therefore, the turning points occurs at x   1 or 3 .We get the turning points by
substituting these values in f(x). The turning points are (-1,8) and (3,-24). The next step is to check which
point is a maximum and which one is a minimum. There are two methods of checking the nature of the
turning points, these are:

17. We can use the second Derivative Test to determine the nature of these points as follows:

d2y
Using theorem 4.2.1, given above, we find the second derivative as follows;  6x  6
dx 2

Page 107 | H A M A C H I L A . I
d2y
At x = -1,  6 x  6  6(1)  6  12  0 , this implies that at (-1, 8), the function has a relative
dx 2
d2y
Maximum. At x = 3,  6 x  6  6(3)  6  18  6  12  0 , this implies that at ( 3, -24), the
dx 2
function has a minimum.

18. We can determine the nature of the turning points by using the gradient method as follows;

A local maximum occurs when y' = 0 and y' changes sign from positive to negative (as we go left to right)
and

A local minimum occurs when y' = 0 and y' changes sign from negative to positive.

dy
(i) So, we take the point (-1,8) and the first derivative is-  3x 2  6 x  9 , we take a point to the
dx
left of the critical value x = -1, which is x = -2, and substitute in the first derivative as;
dy
 3x 2  6 x  9  3(2) 2  6(2)  9  12  12  9  15 , implies the gradient is Positive.
dx

Then take another value of x on the right of the critical value, say 0. Substitute it in the first derivative as :
dy
 3x 2  6 x  9  3(0) 2  6(0)  9  9 , implies the gradient is negative. This means the function is
dx
increasing up to (-1,8) and then decrease after this point. That is the point is a relative maximum point.

(ii) Now we take the point (3,-24), we take appoint to the left of the value x = 3, say, 0. We substitute
dy
this value x = 0 in the first derivative as follows:  3x 2  6 x  9  3(0) 2  6(0)  9  9  0 , implies
dx
the gradient is negative.

Then, we take a value to the right of x = 3, say x = 4, and substitute it in the first derivative as follows;
dy
 3x 2  6 x  9  3(4) 2  6(4)  9  15  0 , implies the gradient is Positive.
dx

This means the function is decreasing up to (3,-24) and then increases. That is, the point (3,-24) is a relative
Minimum point.

The shape of the graph of this function, would look like Figure 4.3.1 given below.

Page 108 | H A M A C H I L A . I
Tangents & Normals
Tangents
The gradient of the tangent to the curve y = f(x) at the point (x1, y1) on the curve is given by: the value of
dy/dx, when x = x1 and y = y1

Normal
Two lines of gradients m1, m2 respectively are perpendicular to each other if the product, m 1x m 2 = -1
Equation of a tangent The equation of a tangent is found using the equation for a straight line of gradient
m, passing through the point (x1, y1) y  y1  m( x  x1 ) . To obtain the equation we substitute in the values

dy
for x1 and y1 and m ( ) and rearrange to make y the subject.
dx

Page 109 | H A M A C H I L A . I
Example: 8.8.1 Find the equation of the tangent to the curve y  2x at the point (1,2).
2

to the function y  2x
2
Solution: The gradient of the tangent line at the point (1,2) is :

dy
m  4 x  4(1)  4
dx

Using the general equation of a straight line, y  y1  m ( x  x1 ) , where ( x1 , y1 ) is a point (1,2). We

have

y  y1  m( x  x1 )
y  2  4( x  1)
y  2  4x  4
y  4x  6
y  4x  6  0

This is the equation of the tangent line to the function y  2x at the point (1,2).
2

Equation of a normal

The equation of a normal is found in the same way as the tangent. The gradient (m 2) of the normal is
1
calculated from; m1m2  1 (where m 1 is the gradient of the tangent) so m2  
m1

Example: 8.8.2
Find the equation of the normal to the curve: y = x2 + 4x + 3, at the point (-1,0).
dy
Therefore gradient ( m1 ) , for the function y  x  4 x  3 is;  2 x  4 , at the point
2
Solution:
dx
dy
(-1, 0), implies you substitute these values in the first derivative as; m1   2 x  4  2(1)  4  2 that
dx
means the gradient of this function at the point (-1,0) is 2.

Let the gradient of then normal line be ( m2 ) , we know that the product of the tangent and normal gradients

is linked by the following relationship: m1 .m2  1

Page 110 | H A M A C H I L A . I
But we know m1  2 , therefore,

2m2  1
1
m2  
2

Using the general equation of a straight line, y  y1  m2 ( x  x1 ) , where ( x1 , y1 ) is a point (-1,0).

1
y  0   ( x  (1))
2
1
y   ( x  1)
2
1 1
So our normal line is y   x 
2 2
2 y  x 1
2y  x 1  0

This is the equation of the line which is normal to a tangent line at the point (-1,0).

Related Rates
dy
You have seen how the chain rule can be used to find implicitly. Another important use of the chain
dx
rule is to find the rates of change of two or more related variables that are changing with respect to time.
For example, when water is drained out of a conical tank, the figure below; the volume V, the radius r, and
the height h of the water level are all functions of time t. Knowing that these variables are related by the

equation V r 2h
3

Page 111 | H A M A C H I L A . I
You can differentiate implicitly with respect to t to obtain the related rate equation

dV   2 dh dr 
 r  h( 2r ) 
dt 3  dt dt 
 2 dh dr 
 r  2rh 
3  dt dt 

From this equation you can see that the rate of change of V is related to the rates of change of both h and r.
Finding the Related Rate. In the conical tank shown above, suppose that the height is changing at a rate of
-0.2 cm per minute and the radius is changing at the rate of -0.1 cm per minute. What is the rate of change
in the volume when the radius is r = 1cm and the height is h = 2 cm? Does the rate of change in volume
depend on the values of r and h?

Guidelines for solving related rate problems

19. Identify all given quantities and quantities to be determined. Make a sketch and label the quantities.

20. Write an equation involving the variables whose rates of change either are given or are to be
determined.

21. Using the chain rule, implicitly differentiate both sides of the equation with respect to time t.

22. After completing step 3, substitute into the resulting equation all known values for the variables and
their rates of change. Then solve for the required rate of change.

Example: 6.9.1

Air is being pumped into a spherical balloon at a rate of 4.5 cubic cm per minute. Find the rate of change
of the radius when the radius is 2 cm.

Solution Let V be the volume of the balloon and let r be its radius. Since the volume is increasing at a
dV 9
rate of 4.5 cubic cm per minute, you know that at time t the rate of change of the volume is  .
dt 2

dV 9 dr
Now the problem can be stated as follows: Given rate:  ( constant rate ). Find; when r = 2
dt 2 dt

To find the rate of change of the radius, you must find an equation that relates the radius r to the volume V.
4
V  r 3 (Volume of sphere ). Implicit differentiation with respect to t produces
3

Page 112 | H A M A C H I L A . I
dV dr
 4r 2 (Differentiation with respect to t )
dt dt

dr 1 dV dr
 ( ) ( Solve for )
dt 4r 2 dx dt

Finally, when r = 2, the rate of change of the radius is

dr 1 9
    0.09 cm per min ute
dt 16  2 

In this example note that the volume is increasing at a constant rate but the radius is increasing at a variable
rate. Just because two rates are related does not mean that they are proportional. In this example, the radius
is growing more and more slowly as t increases. Why?

The Chain Rule is a means of connecting the rates of change of dependent variables.

The derivative tells us the rate of change of one quantity compared to another at a particular instant or
point (so we call it "instantaneous rate of change"). This concept has many applications in electricity,
dynamics, economics, fluid flow, population modeling, queuing theory and so on. Wherever a quantity is
always changing in value, we can use calculus (differentiation and integration) to model its behaviour.

In this section, we will be talking about events at certain times, so we will be using Δt instead of the Δx that
we saw in the last section Derivative from first principles.

Note: This section is part of the introduction to differentiation. We learn some (much easier) rules for
differentiating in the next section, Derivatives of polynomial.

Example: 6.9.2 If air is blown into a spherical balloon at the rate of 10 cm3 how quickly will the radius
grow?

Page 113 | H A M A C H I L A . I
Example: 2.9.7 A spherical raindrop is formed by condensation. In an interval of 10 sec. its volume
increases at a constant rate from 0.010mm3 to 0.500mm3 Find the rate at which the surface area of the
raindrop is increasing, when its radius is 1.0mm

Summary

In this unit you learned the following concepts:

dy f ( x  h)  f ( x )
1.  lim is the derivative of y w.r.t.x from first principles.
dx x x h

Page 114 | H A M A C H I L A . I
2. For any functions uv, then

d du dv
(u  v)   derivatives of sum and difference
dx dx dx

d (uv ) dv du
u v product of functions- product rule
dx dx dx

du dv
v u
d u dx dx , rational functions- quotient rule
( ) 2
dx v v

The Chain Rule

d
If h(x) = g[f(x)], then h' ( x )  g[ f ( x)]  g '[ f ( x)] f ' ( x)
dx

Equivalently, if we write y = h(x) =g(u), where u = f(x), then

General power Rule

If the function f is differentiable and h( x)  [ f ( x)] ( n, a real number ), then


n

d
h' ( x )  [ f ( x)]n  n[ f ( x)]n 1 f ' ( x)
dx

We observe that h(x) = g[f(x)], where g ( x)  x , so that, by virtue of the chain rule, we have
n

h' ( x)  g '[ f ( x)] f ' ( x)


 n[ f ( x)]n1 f ' ( x)

n 1 d2y
Since g ' ( x)  nx , y' ' , f ' ' ( x),
dx 2

These are the forms of writing the second derivative of y = f(x)

Guidelines for solving related rate problems

1. Identify all given quantities and quantities to be determined. Make a sketch and label the quantities.

2. Write an equation involving the variables whose rates of change either are given or are to be determined.

Page 115 | H A M A C H I L A . I
3. Using the chain rule, implicitly differentiate both sides of the equation with respect to time t.

4. After completing step 3, substitute into the resulting equation all known values for the variables and their
rates of change. Then solve for the required rate of change.

Increasing and Decreasing Functions

A function f is increasing on an interval if for any two numbers x1 and x 2 in the interval, x1  x 2 implies

f ( x1  f ( x2 ) .

A function f is decreasing on an interval if for any two numbers x1 and x 2 in the interval, x1  x 2 implies

f ( x1  f ( x2 )

Test for Increasing and Decreasing Functions

Let f be a function that is continuous on the closed interval [a,b] and differentiable on the open interval
(a,b).

1. If f ‘ (x) > 0 for all x in (a,b), then f is increasing on [a,b]

2. If f’ (x) < 0 for all x in (a,b) , then f is decreasing on [a,b]

3. If f ‘ (x) = 0 for all x in (a,b), then f is constant on [a,b]

Guidelines for finding intervals on which a Function is Increasing or Decreasing.

Let f be continuous on the interval (a,b). To find the open intervals on which f is increasing or decreasing,
use the following steps.

1. Locate the critical numbers of f in (a,b), and use these numbers to determine test intervals.

2. Determine the sign of f ‘ (x) at one test value in each of the intervals.

3. Use theorem 4.1.1 to decide whether f is increasing or decreasing on each interval.

The First Derivative test

Let c be a critical number of a function f that is continuous on an open interval I containing c. If f is


differentiable on the interval, except possibly at c, then f9c) can be classified as follows:

1. If f ‘ (x) changes from negative to positive at c, then f(c) is a relative minimum of f.

If f ‘ (x) changes from positive to negative at c, then f(c) is a relative maximum of f.

Page 116 | H A M A C H I L A . I
Second Derivative Test

Let f be a function such that f ' (a )  0 and the second derivative exists on an open interval containing a.

d2y
(i) If  0 at x = a, then the point at x = a is a local minimum.
dx 2

d2y
(ii) If  0 at x = a, then the point at x = a is a local maximum
dx 2

d2y
(iii) If  0 at x = a, the Test Fails.
dx 2

Exercise: 6

1. Calculate the following limits

x2 1 3 x x2  x  6  1 x2 
(i) lim (ii). lim (iii) lim (iv) lim 2 
x 1 x  x3 x  8 x  5 
x 2
x 2  3x x 3
 

dy
2. (a) From first principles find of :
dx

23. y=2x+1 (ii) y  x 3  3x 2  2


1 1
y  2x  1 (v) y (vi) y
(iv) x 1 x 1
x 1
(vii) y (viii) y=3
x 1
dy
(b) Find of the following ;
dx

(i) y = -7x6 (ii) y = 3x5 – 1 (iii) y  13x 4  6 x 3  x  1

1 1
(iv) y   x 8  x 4  3x 2 (v) y  x 4  9 x 2  5x , at the point (3,15).
4 2

1 2
(vi) yx 4

x

Page 117 | H A M A C H I L A . I
3. Sketch the following curve by finding intercepts, maxima and minima and points of inflection:

(a) y  x 3  9x

(b) y  x 4  6x 2

(c ) y  x 5  5x 4

4. Find the critical numbers ( if any) and the open intervals on which the function is increasing or
decreasing.

24. y  ( x  1) ( x  3)
2

25. y  x ( x  3)

26. f ( x)  2 x  4 x  3
2

27. f ( x)  x 2  6 x

28. f ( x)  2 x 3  3x 2  12x

x 5  5x
29. f ( x) 
5
1
30. f ( x)  x 3
1

31. f ( x)  5  x  5

x2
32. f ( x) 
x2  9

33. f ( x)  x 3  6 x 2  15
2
34. f ( x)  ( x  1) 3

1
35. f ( x)  x 
x

x 2  2x  1
36. f ( x) 
x 1

Page 118 | H A M A C H I L A . I
1 4
5. Let f ( x)  x  8 x , use the first derivative test to find any relative extrema of the function. Use
4
your graph to verify your answers.

6. Let ( x)  x  cos x , find any points of inflection of the function in the interval 0  x  2 .

7. Find the second derivative of the function

x
(a) y  2 x 2  sin 2 x (b) y  cot x (c) y  (d) y  x tan x
(1  x) 2

8. Find the equation of the tangent and normal to the curve of the equation at the indicated point.
Graph the function, the tangent and the normal lines.

37. y  ( x  3) 3 at (-2,1)

38. y  3 ( x  2) 2 at (3,1)

39. x 2  y 2  20 , at (2, 4)

1 3
9. Find the points on the graph of y  x  x 2  x  1 when the slope is (a) -1 (b) 2 (c) 0
3

10. Sketch the graph of f ( x)  4  x  2

(a) Is f continuous at x = 2?

(b) Is f differentiable at x = 2? Explain

11. Show that the function satisfies the given differential equation.

Function Equation

y  2 sin x  3 cos x y ' ' y  0

10  cos x
y xy ' y  sin x
x

40. The cross section of a 5 meter trough is an isosceles trapezoid with a 2 meter lower base, a 3 meter
upper base , and an altitude of 2 meters. Water is running into the trough at a rate of 1 cubic meter
per minute. How fast is the water level rising when the water level is i meter deep?

Page 119 | H A M A C H I L A . I
41. Find all relative extrema. Use the second derivative test where applicable.

4
42. f ( x)  x  (b) f ( x)  x 4  4 x 3  2 (c) f ( x)  cos x  x , [0,4 ]
x

43. The radius r of a circle is increasing at a rate of 2 centimetres per minute. Find the rate of change
of the area when (a) r = 6 cm and (b) r = 24 cm. (13)

44. A spherical balloon is inflated with gas at the rate of 500cubic cm per minute. How fast is the radius
of the balloon increasing at the instant the radius is (a) 30 cm and (b) 60 cm
45. A all edges of a cube are expanding at a rate of 3cm per second.
46. How fast is the volume changing when each edge is 10 cm
47. How fast is the volume changing when each edge is (i) 1 cm and (ii) 10 cm
48. At a sand and gravel plant, sand is falling off a conveyor and onto a conical pile at a rate of 10
cubic meters per minute. The diameter of the base of the cone is approximately three times the
altitude. At what rate is the height of the pile changing when the pile is 15 meters high? (21)

dy d 2 y d 3 y
18. Find , , for the following functions:
dx dx 2 dx 3

(i) x 2  2 x  2 xy 2  2

x 2 (3x  1)
(ii) y
x4  2

INTEGRATION

1.0 Introduction

The previous topic on differentiation gave the rule of finding the derivative of a function using the power

rule as: if f ( x)  x , where n  R , the derivative written as f ( x)  nx , the term nx n 1 is the derivative
n n 1

while x n is called the anti-derivative.

Page 120 | H A M A C H I L A . I
Notations:

y  f (x) , is a function which may be continuous or discrete.

 - Integral sign

dx – Independent variable being used

 f ( x)dx Integral of the function f (x) with respect to x

In this case f (x) is called the integrand, i.e. the function being integrated is called the integrand.

2.0 Methods of Integration

2.1 Power Rule

A power rule method will apply if the function is of the form x n , n  R .

x n1 x n1
 x dx  n  1  c , where the term n  1 is called the ant-derivative and c is called the constant of
n

integration.

Properties of integration

49. If F is the anti-derivative of f(x) and G is the anti-derivative of g(x), then

  f ( x)  g ( x)dx   f ( x)dx   g ( x)dx  F  G  c


50. If F is the anti-derivative of f(x) and a is a constant, then  af ( x)dx  a  f ( x)dx  aF  c

 (x  2 x)dx
2
Example 1. Find

 (x  2 x)dx   x 2 dx   2 xdx
2
Solution. Using property (a) and (b)

Page 121 | H A M A C H I L A . I
x3 2x 2
  c
3 2
x3
  x2  c
3

 (x  4 x 2  1)dx
3
Example 2 Find

 (x  4 x 2  1)dx   x 3 dx  4 x 2 dx   dx
3
Solution. Using property (a) and (b)

x4 x3
 4  xc
4 3

2.2 Integration by substitution

(a) If the integral is of the form  f ( g ( x)) g ( x)dx , that is, the integrand is a composite function of

g(x) and the derivative of g(x) as part of the integrand, then the method of substitution can be used to find
the integral of the function.

Example 1 Find x x 2  1dx

Solution Let u  x 2  1 , then the integral can be transformed to a simpler integral as follows;

1
1 1 1 3
2
du du 1 1u 1
du  2 x   xdx , substitute these into the integral,  u   u du  c  u2 c
2 2
2 2 2 21 3
1
2
, but u  x 2  1

3
1 2
 x x  1dx  3 ( x  1) 2  c
2
Therefore,

f ( x)
(b) If the integral is of the form  f ( x)
dx that is the integrand is a rational function and the numerator

f ( x)
is the derivative of the denominator, then  f ( x)
dx  ln f ( x)  c

x
Example 2 Find x 2
1
dx

Page 122 | H A M A C H I L A . I
Solution. This integrand can be transformed by changing variable which will make it easy to integrate, and
this is the main idea of transformation. Let u  x 2  1 , taking derivative of u with respect to x, implies
du
du  2 xdx ,   xdx and xdx is the numerator, so the integral becomes
2

x 1 du 1 du 1
x 2
1
dx   .
u 2 2 u
  ln u  c , but u  x 2  1 so the solution become
2

x 1
x 2
1
dx  ln( x 2  1)  c
2
 ln x 2  1  c

2.3 Integration by parts

If the integrand can be decomposed in the form  uv  , where u is a function which is easy to differentiate

and diminishing but not always and v is a function which is easy to integrate, then integration by parts
method can be used to find for integration using the following formulae:

 uv  uv   vu 
This formula is used for integration by parts, however, it is very important to decompose the integrand in
the form as explained above otherwise, the integration will be complicated.

Example 1 Find x x 1 dx

3
( x  1) 2 3
d 2
Solution. Let u  x   1 , dv  x  1dx , on integration v   ( x  1) 2
dx 3 3
2

Using the integration by parts formula, the integral become  uv  uv   vu 


3
2
( x  1) 2 dx
3
2
 x x  1 dx 
3
x ( x  1) 2
 3
3 5
2 4
 x( x  1) 2  ( x  1) 2  c
3 15

Page 123 | H A M A C H I L A . I
1. Integration by partial fractions

In this section we are going to take a look at integrals of rational expressions of polynomials and once again
let’s start this section out with an integral that we can already do so we can contrast it with the integrals that
we’ll be doing in this section.

So, if the numerator is the derivative of the denominator (or a constant multiple of the derivative of the
denominator) doing this kind of integral is fairly simple. However, often the numerator isn’t the derivative
of the denominator (or a constant multiple). For example, consider the following integral.

In this case the numerator is definitely not the derivative of the denominator nor is it a constant multiple of
the derivative of the denominator. Therefore, the simple substitution that we used above won’t
work. However, if we notice that the integrand can be broken up as follows,

then the integral is actually quite simple.

This process of taking a rational expression and decomposing it into simpler rational expressions that we
can add or subtract to get the original rational expression is called partial fraction decomposition. Many
integrals involving rational expressions can be done if we first do partial fractions on the integrand.

So, let’s do a quick review of partial fractions. We’ll start with a rational expression in the form,

Page 124 | H A M A C H I L A . I
where both P(x) and Q(x) are polynomials and the degree of P(x) is smaller than the degree of Q(x). Recall
that the degree of a polynomial is the largest exponent in the polynomial. Partial fractions can only be
done if the degree of the numerator is strictly less than the degree of the denominator. That is important to
remember.

So, once we’ve determined that partial fractions can be done we factor the denominator as completely as
possible. Then for each factor in the denominator we can use the following table to determine the term(s)
we pick up in the partial fraction decomposition.

Factor in Term in partial


denominator fraction decomposition

Notice that the first and third cases are really special cases of the second and fourth cases respectively.

There are several methods for determining the coefficients for each term and we will go over each of those
in the following examples.

Example 1 Evaluate the following integral.

Solution

Page 125 | H A M A C H I L A . I
The first step is to factor the denominator as much as possible and get the form of the partial fraction
decomposition. Doing this gives,

The next step is to actually add the right side back up.

Example 1 Evaluate the following integral.

Solution
The first step is to factor the denominator as much as possible and get the form of the partial fraction
decomposition. Doing this gives,

The next step is to actually add the right side back up.

Now, we need to choose A and B so that the numerators of these two are equal for every x. To do this
we’ll need to set the numerators equal.

Page 126 | H A M A C H I L A . I
Note that in most problems we will go straight from the general form of the decomposition to this step
and not bother with actually adding the terms back up. The only point to adding the terms is to get the
numerator and we can get that without actually writing down the results of the addition.

At this point we have one of two ways to proceed. One way will always work, but is often more
work. The other, while it won’t always work, is often quicker when it does work. In this case both will
work and so we’ll use the quicker way for this example. We’ll take a look at the other method in a later
example.

What we’re going to do here is to notice that the numerators must be equal for any x that we would

choose to use. In particular the numerators must be equal for and

. So, let’s plug these in and see what we get.

So, by carefully picking the x’s we got the unknown constants to quickly drop out. Note that these are
the values we claimed they would be above.

At this point there really isn’t a whole lot to do other than the integral.

Recall that to do this integral we first split it up into two integrals and then used the substitutions,

on the integrals to get the final answer.

Page 127 | H A M A C H I L A . I
Before moving onto the next example a couple of quick notes are in order here. First, many of the integrals
in partial fractions problems come down to the type of integral seen above. Make sure that you can do
those integrals.

There is also another integral that often shows up in these kinds of problems so we may as well give the
formula for it here since we are already on the subject.

It will be an example or two before we use this so don’t forget about it.

Now, let’s work some more examples.

Example 2 Evaluate the following integral.

Solution
We won’t be putting as much detail into this solution as we did in the previous example. The first thing
is to factor the denominator and get the form of the partial fraction decomposition.

The next step is to set numerators equal. If you need to actually add the right side together to get the
numerator for that side then you should do so, however, it will definitely make the problem quicker if
you can do the addition in your head to get,

Page 128 | H A M A C H I L A . I
As with the previous example it looks like we can just pick a few values of x and find the constants so
let’s do that.

Note that unlike the first example most of the coefficients here are fractions. That is not unusual so don’t
get excited about it when it happens.

Now, let’s do the integral.

Again, as noted above, integrals that generate natural logarithms are very common in these problems so
make sure you can do them.

Example 3 Evaluate the following integral.

Solution
This time the denominator is already factored so let’s just jump right to the partial fraction
decomposition.

Page 129 | H A M A C H I L A . I
Setting numerators gives,

In this case we aren’t going to be able to just pick values of x that will give us all the constants. Therefore,
we will need to work this the second (and often longer) way. The first step is to multiply out the right
side and collect all the like terms together. Doing this gives,

Now we need to choose A, B, C, and D so that these two are equal. In other words we will need to set
the coefficients of like powers of x equal. This will give a system of equations that can be solved.

Note that we used x0 to represent the constants. Also note that these systems can often be quite large and
have a fair amount of work involved in solving them. The best way to deal with these is to use some
form of computer aided solving techniques.

Now, let’s take a look at the integral.

Page 130 | H A M A C H I L A . I
In order to take care of the third term we needed to split it up into two separate terms. Once we’ve done

this we can do all the integrals in the problem. The first two use the substitution , the third uses

the substitution and the fourth term uses the formula given above for inverse tangents.

Example 4 Evaluate the following integral.

Solution
Let’s first get the general form of the partial fraction decomposition.

Now, set numerators equal, expand the right side and collect like terms.

Setting coefficient equal gives the following system.

Page 131 | H A M A C H I L A . I
Don’t get excited if some of the coefficients end up being zero. It happens on occasion.

Here’s the integral.

To this point we’ve only looked at rational expressions where the degree of the numerator was strictly less
that the degree of the denominator. Of course not all rational expressions will fit into this form and so we
need to take a look at a couple of examples where this isn’t the case.

Example 5 Evaluate the following integral.

Solution
So, in this case the degree of the numerator is 4 and the degree of the denominator is 3. Therefore, partial
fractions can’t be done on this rational expression.

To fix this up we’ll need to do long division on this to get it into a form that we can deal with. Here is
the work for that.

Page 132 | H A M A C H I L A . I
So, from the long division we see that,

and the integral becomes,

The first integral we can do easily enough and the second integral is now in a form that allows us to do
partial fractions. So, let’s get the general form of the partial fractions for the second integrand.

Setting numerators equal gives us,

Now, there is a variation of the method we used in the first couple of examples that will work here. There
are a couple of values of x that will allow us to quickly get two of the three constants, but there is no
value of x that will just hand us the third.

What we’ll do in this example is pick x’s to get the two constants that we can easily get and then we’ll
just pick another value of x that will be easy to work with (i.e. it won’t give large/messy numbers
anywhere) and then we’ll use the fact that we also know the other two constants to find the third.

Page 133 | H A M A C H I L A . I
The integral is then,

In the previous example there were actually two different ways of dealing with the x2 in the
denominator. One is to treat it as a quadratic which would give the following term in the decomposition

and the other is to treat it as a linear term in the following way,

which gives the following two terms in the decomposition,

We used the second way of thinking about it in our example. Notice however that the two will give identical
partial fraction decompositions. So, why talk about this? Simple. This will work for x2, but what about x3
or x4? In these cases we really will need to use the second way of thinking about these kinds of terms.

Let’s take a look at one more example.

Example 6 Evaluate the following integral.

Page 134 | H A M A C H I L A . I
Solution
In this case the numerator and denominator have the same degree. As with the last example we’ll need
to do long division to get this into the correct form. I’ll leave the details of that to you to check.

So, we’ll need to partial fraction the second integral. Here’s the decomposition.

Setting numerator equal gives,

Picking value of x gives us the following coefficients.

The integral is then,

Integrals Involving Roots


Now we discuss a technique of integration which can be useful for some integrals with roots in them. Some
integrals with roots can be done quickly with a simple method of substitution and some can be done with
trig substitutions. It is worth to note that not all integrals with roots will allow us to use one of these methods.
The key issue when you are integrating an integral with a root in the denominator, it is very important to

Page 135 | H A M A C H I L A . I
remove the root first by transformation. The choice of the transformation should be such that the root is
removed, otherwise, the integral will be complicated.

Example 1 Evaluate the following integral.

Solution
Note that we have a cube root in the denominator, and the first thing we do is to transform the integrand so
that the cube root is removed.

So, we let u 3  x  3  3u du  dx , Since we have x in the numerator, we make x the subject in our
2

transformation equation and that is x  u 3  3


We now substitute in the original integral. We obtain

x2 u3  3  2
 dx    3u 2 du
3
x3 3
u 3

u 3  5 2
 3  u du
 u 
 3 (u 3  5)udu
 3 (u 4  5u )du
u5 u2
 3[ 5 ] c
5 2
But u 3  x  3  u  3 x  3 , so we substitute in the solution
5 2

x2 ( x  3) 3 ( x  3) 3
 3
x3
dx  3[
5
]  15
2
c

Note if an integral contains the root n f ( x) the substitution which can simplify the integral into a form that

we can handle without difficulties is u  f (x). Otherwise, the integral will be complicated.
n

2
Example 2 Evaluate the following integral  x3 x  10
dx

Solution: We need to transform this integral by removing the square root as, let u 2  x  10  2udu  dx
,

Page 136 | H A M A C H I L A . I
And solving for x we have x  u 2  10 , substitute in the original integral, to obtain;
2 2
 x3 x  10
dx  
u  10  3 u 2
2
2udu

4u
 2 du
u  3u  10
We can express the integrand into partial fraction as follows;
This integral can now be done with partial fractions.

Setting numerators equal gives,

Picking value of u gives the coefficients.

The integral is then,

We have seen how to eliminate roots from the integral and put it into a form that we can handle with no
difficulties. Note however, that this won’t always work and sometimes the new integral will be just as
difficult to do if your choice is not correct.

Page 137 | H A M A C H I L A . I
2.5 Special functions

There are two special function that plays an important role in calculus and these are:

2.5.1 Exponential function e x

This is the only exponential function in the family of exponential functions which is used as a special
function in calculus because of it property of where the derivative is equal to its anti-derivative.

e dx  e x  c
x

1 ax
e dx  e  c , where a is a constant
ax

1 ax1
e
ax 1
dx  e  c , where a is a constant
a

2.5.2 Natural Logarithm function: logex  ln x

When integration certain function involving natural logarithm, the following properties are used:

1
 x dx  ln x  c
1
 x  1dx  ln x  1  c
1 1
 ax  1 dx  a ln ax  1  c , where a is a constant
Integration may include any combinations of the functions given above. What is important in this process
of integration is to check the integrand and identify the right method of integration.

 xe
x
Example 1 Find dx

Solution The integrand has two functions and a method of integration by parts is applicable in this
dv
problem. Let u  x  du  dx , and  e x on integrating v  e x . Substituting in the formula
dx

 uv  uv   vu 
Page 138 | H A M A C H I L A . I
 xe dx  xe   e
x x x

 xe x  e x  c

Integration of Trigonometric functions

Differential notation enables us to express integral formulas in a useful short hand.

d
cos x 
dx
(sin x) , on integrating both sides with respect to x, we get  cos xdx  sin x  c and
d
sin x  ( cos x) , on integrating both sides with respect to x, we get  sin xdx   cos x  c
dx

The rest of the other four trigonometric functions are a combination of cosine and sine functions, such as:
sin x cos x 1 1
tan x  , cot x  , sec x  and csc x 
cos x sin x cos x sin x

Considering integration as ant-derivative we have

Example 1 Find  cos 3xdx


Solution: We can use the method of substitution so that we integrate cos u , were we substitute x by u.

du
Let u  3x , that means du  3dx   dx . Then our integral become
3

du
 cos 3xdx   cos u. 3

Page 139 | H A M A C H I L A . I
1
3
 cos udu

1
 sin u  c
3

1
But u  3x , so our solution is  cos 3xdx  3 sin 3x  c

Example 2 Find  sin( 7 x  2)dx


Solution: We can use the method of substitution so that we integrate sin u , were we substitute x by u.

du
Let u  7 x  2 , that means du  7 dx   dx . Then our integral become
7

du
 sin( 7 x  2)dx   cos u. 7
1
7
 cos udu

1
  cos u  c
7

1
But u  (7 x  2) , so our solution is  sin( 7 x  2)dx   3 cos(7 x  2)  c
Integration using identities

Certain integrals can be evaluated by using trigonometric identities. In this method we seek an identity
which makes it easy to integrate a particular integral.

 tan
2
Example 1 Find xdx

Solution: In this example it is not easy to integrate this integrand, unless if we substitute it with a known
identity which is easy to integrate. We know of an identity which involve tan 2 x . That is
tan 2 x  1  sec 2 x in making tan 2 x the subject we have tan 2 x  sec 2 x  1

 
That means tan xdx  (sec x  1)dx , that is;
2 2

Page 140 | H A M A C H I L A . I
 tan xdx   (sec 2 x  1)dx
2

  sec 2 xdx   dx
 tan x  x  c

 cos
2
Example 2 Find xdx

Solution: In this example it is not easy to integrate this integrand, unless if we substitute it with a known
identity which is easy to integrate. We know of an identity which involve cos 2 x . That is
cos 2 x  1
cos 2 x  . So our integral become
2

1
 cos xdx 
2
(cos 2 x  1)dx
2

1
 [  cos 2 xdx   dx
2
1 1
 [ sin 2 x  x]  c
2 2
1 x
 sin 2 x   c
4 2

cos 2 x
Example 3 Find  sin 3 2 xdx
Solution: This problem can be solved by substitution by letting u  sin 2 x to give us du  2 cos 2 xdx ,
du
that implies  cos 2 xdx , substituting in the original integral, we obtain
2

cos 2 x 1 du
 sin3
x
dx   3 .
u 2
1 3
  u du
2
1 u 2
 c
2 (2)
1
  2 c
4u

cos 2 x 1
But u  sin 2 x , that means our solution is  sin 3
2x
dx  
4 sin 2 x
c

Page 141 | H A M A C H I L A . I

Find 16 x sin (2 x  1) cos(2 x  1)dx
3 2 2
Example 4

Solution: We use the method of substitution by letting u  2 x 2  1 , on differentiating with respect to x, we


du
have du  4 xdx   xdx , on substituting to the original integral, we obtain
4

du
16 x sin (2 x 2  1) cos(2 x 2  1)dx   16 sin 3 u cos u
3

4
 4 sin 3 u cos udu

We let p  sin u  dp  cos udu so our integral become

p4
4 sin 3 u cos udu  4 p 3 dp  4 c
4

But we know p  sin u and u  2 x 2  1 , so our solution is;

16 x sin (2 x 2  1) cos(2 x 2  1)dx  sin 4 u  c


3

 sin 4 (2 x 2  1)  c

Integral Formulas

 sec xdx  tan x  c


2
51.

52. csc 2 xdx   cot x  c

53.  sec x tan xdx  sec x  c


54.  csc x cot xdx   csc x  c

Page 142 | H A M A C H I L A . I
Definite integral

In the previous chapter effort was focus on understanding what is integration and methods of integration.

The integrals used of the form  f ( x)dx is called an indefinite integral. Indefinite integrals carry a constant
of integration that is why all integrals obtained in the previous section has a constant of integration c. In
this section efforts will be made to explain what definite integrals are.

There is also a little bit of terminology to be noted here. The number “a” that is at the bottom of the integral
sign is called the lower limit of the integral and the number “b” at the top of the integral sign is called the
upper limit of the integral. Also, despite the fact that a and b were given as an interval the lower limit
does not necessarily need to be smaller than the upper limit. Collectively it is often called a and b as the
interval of integration

Fundamental Theorem of Calculus.

If f(x) is a continuous function which is once differentiable in a closed interval [a,b], the integral can be
completely described by the end points of the end points of the closed interval.

 f ( x)dx  F ( x)  F (b)  F (a) , where F(x) is called the anti-derivative, a is the lower limit of the
b
a
a

closed interval and b is the upper limit of the closed interval.

This integral is called definite integral and in this case a value can be found, whereas with indefinite integral
the solution will be a function with a constant of integration to give a family of solutions depending on the
values of c. In evaluating definite integrals fundamental theorem of calculus is used in finding the value of
the integral.

Properties of the definite integral

b a
55.  f ( x)dx   f ( x)dx , Limits can be swapped on any definite integral, but change the sign of
a b

the integral.

a
56.  f ( x)dx  0 , If the upper and lower limits are the same, then the integral is zero.
a

Page 143 | H A M A C H I L A . I
b b
57.  cf ( x)dx  c  f ( x)dx , where c is any number. So, as with limits, derivatives, and indefinite
a a

integrals a constant can factored out.

b b b
58.  [ f ( x) g ( x)]dx   f ( x)dx   g ( x)dx , the integral sign can be distributed over plus and minus
a a a

signs.

b c b
59.  f ( x)dx   f ( x)dx   f ( x)dx , where c is any number.
a a c
This property is more important than

it may be realized at first. One of the main uses of this property is to tell how one can integrate a
function over the adjacent intervals, [a,c] and [c,b]. Note however that c doesn’t need to be between
a and b.

b b
60. 
a
f ( x)dx   f (t )dt , The point of this property is to notice that as long as the function and limits
a

are the same the variable of integration that is being used in the definite integral won’t affect the
answer.

Example 1. Using the properties given above evaluate each of the following
0

 (x  2)dx
2
61.
2

Solution
0 2

 (x  2)dx    ( x 2  2)dx
2

2 0

x3
 [(  2 x)]02
3
8
 [(  4)
3
4

3
2

 (5 x  5 x)dx
2
62.
1

Solution: For this part notice that 5 can be factored out of both terms and then out of the integral using the
third property.

Page 144 | H A M A C H I L A . I
2 2

 (5 x  5 x)dx  5 ( x 2  x)dx
2

1 1
2 2
 5 x 2 dx  5 xdx
1 1
3
x 2 x2 2
5 1 5 1
3 2
8 1 4 1
 5(  )  5  
3 3 2 2
7 3
 5( )  5( )
3 2
35 15
 
3 2
70  45

6
115

6

 (t  2)dt
2
(c)
0

Solution: In this case the only difference is the letter used and so this is just going to use property 6.
2
t3
 (t  2)dx  [  2t ]02
2

0
3
8
 [  4]  0
3
4

3
10 6
Example 2. Given that 6
f ( x)dx  23 and  g ( x)dx  9 determine the value of
10

 (2 f ( x)  10g ( x))dx
10

Solution: First use the fourth property to break up the integral and the third property to factor the constant
6 6 6

10
 (2 f ( x)  10g ( x))dx  2  f ( x)dx  10  g ( x)dx
10 10

Page 145 | H A M A C H I L A . I
Now notice that the limits on the first integral are interchanged with the limits on the given integral so
switch them using the first property above (and adding a minus sign of course). Once this is done the
known values can be plugged in the integrals.

10 10 5
Example 3 Given that  f ( x)dx  6 ,  f ( x)dx  2 , and  f ( x)dx  4
12 100 100
determine the value of

12

 f ( x)dx .
5

Solution: To solve this problem property number 5 and use of property 1 is required. What is required here
is to figure out how to correctly break up the integral using property 5 in order to use the given pieces of
information. Note that there is an integral with -5 as one of the limits. It is not a lower limit, hence property
1 can be applied. The other limit is 100 so this is the number c in order to use property 5. So the integral
12 100 12
can be decomposed as:  f ( x)dx   f ( x)dx   f ( x)dx
5 5 100

The value of the first integral can be found, but the second still isn’t in the list of know integrals. However,
the second has a limit of 100 in it. The other limit for this second integral is -10 and this will be c in this
application of property 5.

Now property 1 can be used on the first and third integrals to get the limits to match up with the known
integrals. After that plug in for the known integrals

Page 146 | H A M A C H I L A . I
2
1
Example 4 Evaluate  (x  x
4
2
)dx

2 2 2
1 1
Solution: The given integral can be written as  ( x  2 )dx   xdx   2 dx
4 x 4 4 x

On integrating term by term this yield

2 2 2
1 1
4
 (x  x 2
)dx   xdx   2 dx
4 4 x

x2 1 2
[  ]
2 x 4
22 1 (4) 2 1
[  ][  ]
2 2 2 4
1 1
 (2  )  (8  )
2 4
5 31
 
2 4
21

4

There are also some other properties that can used in comparing the general size of definite integrals. Here
they are.

More Properties
b
7.  cdx  c(b  a) , where c is a constant.
a

b
8. If f ( x)  0 , for a  x  b ,, then  f ( x)dx  0
a

b b
9. If f ( x)  g ( x) , for a  x  b , then  f ( x)dx   g ( x)dx
a a

b
10. If m  f ( x)  M , for a  x  b , then m(b  a)   f ( x)dx  M (b  a)
a

b b
11.  f ( x)dx  
a a
f ( x) dx

Page 147 | H A M A C H I L A . I
REFERENCES
Prescribed Textbooks

1. Buckhouse, J.K and Houldsworth, S.P.T.1985.Pure Mathematics 1.Longman

2. Roland E. Larson , Robert P. Hostetler, Bruce H Edward: 1998: Calculus with Analytical Geometry.
Houghton Mifflin Co. Boston. New York

3. S.T.Tan: 2004: College Mathematics .Thomson,Books/cole

4. www.mathstutor.com.

Page 148 | H A M A C H I L A . I

You might also like